deon thi thu dai hoc

48 8 0
deon thi thu dai hoc

Đang tải... (xem toàn văn)

Tài liệu hạn chế xem trước, để xem đầy đủ mời bạn chọn Tải xuống

Thông tin tài liệu

Read the following passage and mark the letter A, B, C or D on your answer sheet to indicate the correct answer to each of the following questionsC. Lead poisoning in children is a majo[r]

(1)

TRƯỜNG THPT LÊ HỒNG PHONG KỲ THI THỬ ĐẠI HỌC - LẦN II Thị xã Bỉm Sơn NĂM HỌC 2010-2011

Môn thi: Tiếng Anh

Thời gian làm bài: 90 phút ĐỀ CHÍNH THỨC

(đề thi gồm trang) Mã đề: 121

Họ tên thí sinh: Số báo danh:

I Mark the letter A, B, C, or D on your answer sheet to indicate the word that differs from the rest in the position of the main stress in each of the following sentences.

01.A. competitive B. pedestrian C. catalogue D. experienced

02.A. government B. insurance C. envelope D. lavatory

03.A. preferably B. screwdriver C. fortunately D. imprudent

04.A. police B. hotel C. injured D. cassette

05.A. mistake B. nuclear C. machine D. refer

II Read the following passage and mark the letter A, B, C or D on your answer sheet to indicate the correct answer to each of the following questions.

Lead poisoning in children is a major health concern Both low and high doses of lead can have serious effects Children exposed to high doses of lead often suffer permanent nerve damage, mental retardation, short attention spans, distractibility, poor academic performance, and behavioural problems This is not a new concern As early as 1904, lead poisoning in children was linked to lead-based paint Microscopic lead particles from paint are absorbed into bloodstream when the children ingest flakes of chipped paint, plaster or paint dust from suckling, or chewing on toys and other objects painted with lead-based paint Despite amount of lead used in their paint, this source of leading poisoning is still the most common and most dangerous Children living in older, dilapidated houses are particularly at risk

06. Which of the following is NOT true?

A. Lead poisoning can lead to mental retardation

B. Only high dose of lead can have serious effects

C. American paint companies today must comply with strict regulations regarding the amount of lead used in their paint

D. Lead poisoning in children was linked to lead-based paint in the 1900s

07. According to the passage, what is the most common source of lead poisoning in children?

A. lead-based paint B. household dust C. dilapidated houses D. painted toys

08. Which of the following is closest in meaning to the phrase "at risk" ?

A. in safe way B. in danger C. no harm D. in good condition

09. Which of the following is closest in meaning to the word "suffer" ?

A. feel paint from B. reveal C. experience D. grieve with

10. Which of the following is closest in meaning to "chipped"?

A. unhealthy B. sprayed C. canned D. fragmented

11. The phrase "exposed to" could be best replaced by which of the following?

A. displaying B. conducting C. familiar with D. in contact with

12. What does the author imply in the final sentence of the passage?

A. Old homes were painted with lead-based paint

B. Lead-based paint chips off more easily than newer paints

C. Old homes need to be rebuilt in order to be safe for children

D. Poor people did not comply with the regulations

13. The word " dilapidated " is closest in meaning to which of the following?

A. broken down B. poorly painted C. fell down D. unpainted

14. Which of the following can be inferred from the passage?

(2)

B. Paint companies have always followed regulations regarding the amount of lead used in their paint

C. Paint companies aren't required to limit the amount lead used in their paint

D. Paint companies must limit the amount lead used in their paint

15. What is the main topic of the passage?

A. Lead poisoning in children B. Problems with household paint

C. Major health concern for children D. Lead paint in older houses

III.Mark the letter A, B, C, or D on your answer sheet to show the underlined part that needs correction

16. Vitamin K providing the necessary impetus for the synthesis of at least two proteins involved in blood clotting

A. providing B. necessary C. at least D. involved

17. For the past few years, researchers have perfecting their control over the movements of cells and microbes by using low power laser beams

A. using B. have perfecting C. their D. few years

18. The nests of most bird species are strategic placed to camouflage them against predators

A. species B. against predators C. most D. strategic

19. Perhaps the most unique thing about carbon atoms are their ability to combine with themselves

A. are B. about C. the most D. to combine

20. Organisms and their cells live by maintaining a constant exchange of elemental, ions, minerals and gases

A. gases B. of elemental C. by maintaining D. constant

IV Read the following passage and mark the letter A, B, C or D on your answer sheet to indicate the correct answer to each of the following questions.

Watching for wildlife in the forests, we rarely see past the surface of things Standing on the ground floor, we scan the leafy rafters, entirely overlooking the living world in the soil beneath our feet The forest's basement is a secret world As different from our own world as water is from air, the soil seems quiet, even dead But life bustles down below: a cubic inch of top soil may contain billions of creatures Predators and prey roam beneath as well as above the forest floor Furthermore, those upstairs and downstairs forest denizens live closed linked lives Soil-dwelling bacteria and fungi break down dead organic matter into molecules that above ground plants use for food Those plants as well as animals, mature and die, leaving more organic matter to fuel the folks downstairs

Like a well-insulated house, the soil protects its tenants from extreme temperatures, and from rain and snow It also provides a bulwark against predators that roam the surface world But the dense, protecting soil also limits mobility Soil creatures must be specially equipped in order to travel easily through their dark, constricting realm Earthworms and ants are the champion earth-movers, creating channels that allow air and water to enter the soil While ants travel relatively far from their nests, earthworms work small areas, reprocessing vast amounts of soil into fertile "waste" In a single year, as much as 36 tons of soil may pass through the alimentary tracts of all the earthworms living in an acre of soil

21. The word " bulwark" in the passage is closest in meaning to

A. radar B. tomb C. barrier D. gateway

22. The word "linked" in the passage is closest in meaning to

A. mechanical B. chained C. related D. measured

23. The word "constricting" in the passage is closest in meaning to

A. limiting B. damp C. heavy D. deep

24. The word "reprocessing" in the passage is closest in meaning to

A. converting B. eliminating C. transporting D. arranging

25. The main topic of the passage is

A. the life cycle of ants and worms B. life in the forest soil

C. the habits of the forest animals D. a description of a forest scene

26. The word "bustles" in the passage is closest in meaning to

(3)

27. It could be expected that ants

A. are more mobile than earthworms B. live only above ground

C. perform similar functions to fungi D. move more earth than earthworms

28. It can be inferred from the passage that the forest soil is

A. sparely inhabited B. unknown to man C. densely inhabited D. devoid of life

29. The soil offers creatures that live underground protection EXPECT

A. bad weather B. extreme heat and cold C. enemies D. bacteria and fungi

30. According to the passage, what is the main function of bacteria and fungi?

A. to provide food for plant life B. to build walls in the soil

C. to kill mature plants D. to help aerate the soil

V Mark the letter A, B, C or D on your answer sheet to indicate the sentence that is closest in meaning to each of following sentences.

31. This is the most delicious meat I've ever eaten

A. Not ever in my life have I had such a wonderful meat

B. Never in my life have I had such a delicious meat

C. At no time in my life have I never tasted this excellent meat

D. I have never eaten such more delicious meat

32. She started work three months ago

A. It is three months since she started work B. She is working here for three months now

C. She had been working for three months D. It's three months that she worked for

33. My father finds maps hard to follow

A. My father always gets lost B. My father can't read maps at all

C. Map-reading is not interesting to my father at all D. My father has trouble following maps

34. She has lost her appetite recently

A. Her appetite has been very good B. She has gone off food recently

C. She has eaten a lot of food recently D. She hasn't had any food recently

35. We came home early to avoid the coming storm

A. In order to avoiding the coming storm, we came home early

B. Because it was predicted storming, we decided to come home earlier than unusual

C. We didn't come home late to not get the coming storm

D. We came home early in order to avoid the coming storm

36. My career as a teacher began 14 years ago

A. For 14 years have I been a teacher B. I was a teacher for 14 years

C. I have started teaching for 14 years now D. I have been a teacher for 14 years now

37. This affair does not concern you

A. This affair is no business of yours B. Your concern is to this affair

C. This affair is not interesting D. Don't this affair

38. Not many people attended the meeting

A. People didn't show up at the meeting B. There was a poor turn-out for the meeting

C. Too many people turned out at the meeting D. No people came to the meeting

39. My father regretted working for that company

A. My father didn't like working for that company

B. My father wished he hadn't worked for that company

C. If only my father was not working for that company

D. It is my father's regret working for that company

40. "How beautiful is the dress you have just bought!" Peter said to Mary

A. Peter said thanks to Mary for her beautiful dress

B. Peter asked Mary how she had just bought her beautiful dress

C. Peter complimented Mary on her beautiful dress

(4)

VI Mark the letter A, B, C, or D on your answer sheet to indicate the correct answer to each of the following sentences

41. This film several scenes which are very funny

A. features B. depicts C. illustrates D. pictures

42. The product was withdrawn from sale because there was no longer any for it

A. interest B. order C. call D. claim

43. No one can visit her because she has a very disease

A. corporal B. exasperating C. intoxicating D. contagious

44. She should have been here but she's flu

A. gone down with B. gone through with C. come up against D. come in for

45. By the year 2012, many people currently employed their jobs

A. are losing B. will have lost C. have lost D. will be losing

46. How exactly did you set training the horses to work so well together?

A. empty B. to C. loose D. up

47. There's a rumour that the Nation Bank is going to the company I work for

A. take off B. take on C. take over D. overtake

48. " What's the biggest problem in your country?" - " We have of energy."

A. the shortage B. a shortage C. some shortage D. shortage

49. "Who broke the window?" - " "

A. I am B. I was, thanks C. I did Excuse me D. I did Sorry

50. All the cereal grains grow on the prairies and plains of the United States

A. except the rice B. excepting rice C. but rice D. but for rice

51. "Don't tell anyone my new address" - " "

A. I can't B. I don't C. I wouldn't D. I won't

52. Automobile propane gas emits fewer dangerous pollutants into the atmosphere

A. that are used B. using C. use D. can use

53. He tried to explain how he felt, but he was unable to his true feelings

A. inhibit B. evolve C. articulate D. inflict

54. Scientists are still uncertain of originated millions of years ago

A. which the universeB. how the universe C. with the universe D. about the universe

55. Only when a great deal of more information has been obtained, to plan a manned trip to the planet

A. it will probably be B. it will be possible C. will it possible D. it will be likely

56. As the President was absent, I was ask to the meeting

A. officiate B. chair C. regulate D. govern

57. A child in the first grade tends to be all of the other children in the class

A. the same age as B. as old like C. the same old to D. the same age than

58. I'd like to have a bath, but there doesn't seem to be a or anything to stop

A. cork B. lid C. cap D. plug

59. Bigamy is a situation in which a man two women at the same time

A. is married to B. married C. is marry to D. marries to

60. A dog on his owner's lap may refuse to eat from a bowl on the floor

A. fed B. is fed C. was fed D. to feed

61. I had to get up early, I'd have missed the train

A. yet B. if not C. otherwise D. but

62. "Can I use your phone?" - " "

A. Of course B. You may not C. No, you can't D. You must

63. I'd sooner deliver the new furniture tomorrow

A. mustn't B. wouldn't C. shouldn't D. didn't

64. Technological advances aid in teaching, the basic role of teachers stays the same

(5)

65. the public's concern about the local environment this new road scheme will have to be abandoned

A. As regards B. However much C. In view of D. In the event of

66. - Janet: "Do you feel like going to the cinema tonight?" - Susan: " "

A. I don't agree, I am afraid B. I feel very bored

C. You're welcome D. That would be great

67. will remain the foremost world language is considered inevitable by many people

A. Because English B. That English C. English D. It is English that

68. Their climate is not dry at all; in fact, they have of water

A. a redundancy B. an abundance C. a conglomeration D. a deficiency

69. When another person sneezes, you say " "

A. Bless you B. Great you C. Pardon? D. See you

70. That popular magazine has many

A. prescriptions B. spectators C. subscribers D. denominations VII Read the following passage and mark the letter A, B, C or D on your answer sheet to indicate the correct word for each of the blanks.

In this age of (71) telephone networks and electronic mail, it seems that fewer and even fewer people are taking time to sit down and write letters (72) friends and relatives For hundreds of years, letters were the only way to keep (73) people who were any distance away and letter-writing was seen as an important skill for all learned people (74) Gradually, (75) , the importance of writing letters is decreasing to a point that majority of us have to (76) a special effort to turn out something worthwhile when we apply for a job or make an appointment In business circles the tendency is for routine communications to become shorter .(77) clients may appreciate a detailed letter, an employee who sends out long letter is often regarded as (78) Many people prefer the telephone in all circumstances and its speed is essential in many situations but (79) have you put the telephone down, dissatisfied with what you have managed to say? I don't think I'll throw my (80) away yet

71.A. advanced B. progressive C. all are correct D. highly-developed

72.A. with B. for C. to D. from

73.A. in step with B. on good terms with C. in favour of D. in contact with

74.A. mastered B. mastering C. to master D. to be mastered

75.A. in short B. for example C. therefore D. however

76.A. cause B. make C. D. create

77.A. Even though B. As though C. Despite D. However

78.A. unusual B. inefficient C. unimportant D. impossible

79.A. how much B. how about C. how often D. how long

(6)

Đáp án đề: 121

01 - - } - 21 - - } - 41 { - - - 61 - - } -

02 - | - - 22 - - } - 42 - - } - 62 { - - -

03 - - - ~ 23 { - - - 43 - - - ~ 63 - - } -

04 - | - - 24 { - - - 44 { - - - 64 - - - ~

05 - | - - 25 - | - - 45 - | - - 65 - - } -

06 - | - - 26 { - - - 46 - - - ~ 66 - - - ~

07 { - - - 27 { - - - 47 - - } - 67 - | - -

08 - | - - 28 - - } - 48 - | - - 68 - | - -

09 - - } - 29 - - - ~ 49 - - - ~ 69 { - - -

10 - - - ~ 30 { - - - 50 - - } - 70 - - } -

11 - - - ~ 31 - - - ~ 51 - - - ~ 71 - - } -

12 - - } - 32 { - - - 52 - | - - 72 - - } -

13 { - - - 33 - - - ~ 53 - - } - 73 - - - ~

14 - - - ~ 34 - | - - 54 - | - - 74 - - } -

15 { - - - 35 - - - ~ 55 - - } - 75 - - - ~

16 { - - - 36 - - - ~ 56 - | - - 76 - | - -

17 - | - - 37 { - - - 57 { - - - 77 { - - -

18 - - - ~ 38 - | - - 58 - - - ~ 78 - | - -

19 { - - - 39 - | - - 59 { - - - 79 - - } -

(7)

TRƯỜNG THPT LÊ HỒNG PHONG KỲ THI THỬ ĐẠI HỌC - LẦN II Thị xã Bỉm Sơn NĂM HỌC 2010-2011

Môn thi: Tiếng Anh

Thời gian làm bài: 90 phút ĐỀ CHÍNH THỨC

(đề thi gồm trang) Mã đề: 122

Họ tên thí sinh: Số báo danh:

I Read the following passage and mark the letter A, B, C or D on your answer sheet to indicate the correct word for each of the blanks.

In this age of (1) telephone networks and electronic mail, it seems that fewer and even fewer people are taking time to sit down and write letters (2) friends and relatives For hundreds of years, letters were the only way to keep (3) people who were any distance away and letter-writing was seen as an important skill for all learned people (4) Gradually, (5) , the importance of writing letters is decreasing to a point that majority of us have to (6) a special effort to turn out something worthwhile when we apply for a job or make an appointment In business circles the tendency is for routine communications to become shorter .(7) clients may appreciate a detailed letter, an employee who sends out long letter is often regarded as (8) Many people prefer the telephone in all circumstances and its speed is essential in many situations but (9) have you put the telephone down, dissatisfied with what you have managed to say? I don't think I'll throw my (10) away yet

01.A. progressive B. all are correct C. highly-developed D. advanced

02.A. with B. for C. from D. to

03.A. on good terms with B. in favour of C. in step with D. in contact with

04.A. mastering B. to be mastered C. mastered D. to master

05.A. in short B. for example C. however D. therefore

06.A. cause B. C. create D. make

07.A. Even though B. As though C. Despite D. However

08.A. impossible B. unimportant C. inefficient D. unusual

09.A. how long B. how often C. how much D. how about

10.A. effort B. telephone C. pen D. letter

II Read the following passage and mark the letter A, B, C or D on your answer sheet to indicate the correct answer to each of the following questions.

Lead poisoning in children is a major health concern Both low and high doses of lead can have serious effects Children exposed to high doses of lead often suffer permanent nerve damage, mental retardation, short attention spans, distractibility, poor academic performance, and behavioural problems This is not a new concern As early as 1904, lead poisoning in children was linked to lead-based paint Microscopic lead particles from paint are absorbed into bloodstream when the children ingest flakes of chipped paint, plaster or paint dust from suckling, or chewing on toys and other objects painted with lead-based paint Despite amount of lead used in their paint, this source of leading poisoning is still the most common and most dangerous Children living in older, dilapidated houses are particularly at risk

11. What is the main topic of the passage?

A. Lead paint in older houses B. Major health concern for children

C. Problems with household paint D. Lead poisoning in children

12. What does the author imply in the final sentence of the passage?

A. Old homes need to be rebuilt in order to be safe for children

B. Lead-based paint chips off more easily than newer paints

C. Poor people did not comply with the regulations

D. Old homes were painted with lead-based paint

13. Which of the following can be inferred from the passage?

(8)

B. Paint companies can no longer use lead in their paint

C. Paint companies must limit the amount lead used in their paint

D. Paint companies aren't required to limit the amount lead used in their paint

14. According to the passage, what is the most common source of lead poisoning in children?

A. dilapidated houses B. lead-based paint C. household dust D. painted toys

15. The word " dilapidated " is closest in meaning to which of the following?

A. poorly painted B. unpainted C. broken down D. fell down

16. Which of the following is closest in meaning to "chipped"?

A. canned B. sprayed C. fragmented D. unhealthy

17. The phrase "exposed to" could be best replaced by which of the following?

A. displaying B. in contact with C. familiar with D. conducting

18. Which of the following is closest in meaning to the phrase "at risk" ?

A. in safe way B. no harm C. in good condition D. in danger

19. Which of the following is closest in meaning to the word "suffer" ?

A. experience B. grieve with C. feel paint from D. reveal

20. Which of the following is NOT true?

A. Lead poisoning can lead to mental retardation

B. American paint companies today must comply with strict regulations regarding the amount of lead used in their paint

C. Lead poisoning in children was linked to lead-based paint in the 1900s

D. Only high dose of lead can have serious effects

III Mark the letter A, B, C, or D on your answer sheet to indicate the correct answer to each of the following sentences

21. Technological advances aid in teaching, the basic role of teachers stays the same

A. but B. with C. because of D. despite

22. the public's concern about the local environment this new road scheme will have to be abandoned

A. However much B. As regards C. In the event of D. In view of

23. He tried to explain how he felt, but he was unable to his true feelings

A. inflict B. articulate C. evolve D. inhibit

24. Only when a great deal of more information has been obtained, to plan a manned trip to the planet

A. it will be likely B. will it possible C. it will be possible D. it will probably be

25. Their climate is not dry at all; in fact, they have of water

A. a deficiency B. a redundancy C. a conglomeration D. an abundance

26. "Who broke the window?" - " "

A. I was, thanks B. I am C. I did Excuse me D. I did Sorry

27. "Don't tell anyone my new address" - " "

A. I don't B. I can't C. I wouldn't D. I won't

28. When another person sneezes, you say " "

A. Bless you B. Great you C. See you D. Pardon?

29. I'd like to have a bath, but there doesn't seem to be a or anything to stop

A. cap B. cork C. lid D. plug

30. How exactly did you set training the horses to work so well together?

A. empty B. to C. loose D. up

31. This film several scenes which are very funny

A. illustrates B. features C. depicts D. pictures

32. By the year 2012, many people currently employed their jobs

A. are losing B. have lost C. will be losing D. will have lost

33. Bigamy is a situation in which a man two women at the same time

(9)

34. A dog on his owner's lap may refuse to eat from a bowl on the floor

A. is fed B. fed C. was fed D. to feed

35. I had to get up early, I'd have missed the train

A. if not B. otherwise C. yet D. but

36. As the President was absent, I was ask to the meeting

A. officiate B. regulate C. chair D. govern

37. There's a rumour that the Nation Bank is going to the company I work for

A. overtake B. take off C. take on D. take over

38. Automobile propane gas emits fewer dangerous pollutants into the atmosphere

A. can use B. that are used C. using D. use

39. will remain the foremost world language is considered inevitable by many people

A. Because English B. That English C. It is English that D. English

40. No one can visit her because she has a very disease

A. corporal B. contagious C. exasperating D. intoxicating

41. Scientists are still uncertain of originated millions of years ago

A. with the universe B. which the universe C. how the universe D. about the universe

42. - Janet: "Do you feel like going to the cinema tonight?" - Susan: " "

A. I feel very bored B. You're welcome

C. That would be great D. I don't agree, I am afraid

43. All the cereal grains grow on the prairies and plains of the United States

A. but rice B. excepting rice C. but for rice D. except the rice

44. I'd sooner deliver the new furniture tomorrow

A. wouldn't B. mustn't C. shouldn't D. didn't

45. A child in the first grade tends to be all of the other children in the class

A. the same old to B. the same age than C. as old like D. the same age as

46. The product was withdrawn from sale because there was no longer any for it

A. interest B. call C. order D. claim

47. "Can I use your phone?" - " "

A. No, you can't B. You must C. Of course D. You may not

48. " What's the biggest problem in your country?" - " We have of energy."

A. a shortage B. some shortage C. shortage D. the shortage

49. She should have been here but she's flu

A. come in for B. gone through with C. gone down with D. come up against

50. That popular magazine has many

A. prescriptions B. spectators C. denominations D. subscribers

IV Mark the letter A, B, C, or D on your answer sheet to indicate the word that differs from the rest in the position of the main stress in each of the following sentences.

51.A. mistake B. machine C. refer D. nuclear

52.A. screwdriver B. preferably C. imprudent D. fortunately

53.A. injured B. hotel C. cassette D. police

54.A. envelope B. insurance C. government D. lavatory

55.A. experienced B. catalogue C. competitive D. pedestrian

V Mark the letter A, B, C, or D on your answer sheet to show the underlined part that needs correction.

56. Vitamin K providing the necessary impetus for the synthesis of at least two proteins involved in blood clotting

A. providing B. involved C. necessary D. at least

57. For the past few years, researchers have perfecting their control over the movements of cells and microbes by using low power laser beams

A. using B. their C. have perfecting D. few years

(10)

A. most B. species C. strategic D. against predators

59. Perhaps the most unique thing about carbon atoms are their ability to combine with themselves

A. the most B. are C. to combine D. about

60. Organisms and their cells live by maintaining a constant exchange of elemental, ions, minerals and gases

A. constant B. by maintaining C. gases D. of elemental

VI Mark the letter A, B, C or D on your answer sheet to indicate the sentence that is closest in meaning to each of following sentences.

61. This is the most delicious meat I've ever eaten

A. At no time in my life have I never tasted this excellent meat

B. Not ever in my life have I had such a wonderful meat

C. Never in my life have I had such a delicious meat

D. I have never eaten such more delicious meat

62. Not many people attended the meeting

A. No people came to the meeting B. People didn't show up at the meeting

C. Too many people turned out at the meeting D. There was a poor turn-out for the meeting

63. She started work three months ago

A. She had been working for three months B. It is three months since she started work

C. She is working here for three months now D. It's three months that she worked for

64. My father finds maps hard to follow

A. Map-reading is not interesting to my father at all B. My father always gets lost

C. My father can't read maps at all D. My father has trouble following maps

65. We came home early to avoid the coming storm

A. Because it was predicted storming, we decided to come home earlier than unusual

B. In order to avoiding the coming storm, we came home early

C. We came home early in order to avoid the coming storm

D. We didn't come home late to not get the coming storm

66. This affair does not concern you

A. This affair is not interesting B. This affair is no business of yours

C. Your concern is to this affair D. Don't this affair

67. "How beautiful is the dress you have just bought!" Peter said to Mary

A. Peter complimented Mary on her beautiful dress

B. Peter promised to buy Mary a beautiful dress

C. Peter asked Mary how she had just bought her beautiful dress

D. Peter said thanks to Mary for her beautiful dress

68. My career as a teacher began 14 years ago

A. For 14 years have I been a teacher B. I have started teaching for 14 years now

C. I have been a teacher for 14 years now D. I was a teacher for 14 years

69. My father regretted working for that company

A. It is my father's regret working for that company

B. My father didn't like working for that company

C. If only my father was not working for that company

D. My father wished he hadn't worked for that company

70. She has lost her appetite recently

A. She has gone off food recently B. She hasn't had any food recently

C. Her appetite has been very good D. She has eaten a lot of food recently

VII Read the following passage and mark the letter A, B, C or D on your answer sheet to indicate the correct answer to each of the following questions.

(11)

forest's basement is a secret world As different from our own world as water is from air, the soil seems quiet, even dead But life bustles down below: a cubic inch of top soil may contain billions of creatures Predators and prey roam beneath as well as above the forest floor Furthermore, those upstairs and downstairs forest denizens live closed linked lives Soil-dwelling bacteria and fungi break down dead organic matter into molecules that above ground plants use for food Those plants as well as animals, mature and die, leaving more organic matter to fuel the folks downstairs

Like a well-insulated house, the soil protects its tenants from extreme temperatures, and from rain and snow It also provides a bulwark against predators that roam the surface world But the dense, protecting soil also limits mobility Soil creatures must be specially equipped in order to travel easily through their dark, constricting realm Earthworms and ants are the champion earth-movers, creating channels that allow air and water to enter the soil While ants travel relatively far from their nests, earthworms work small areas, reprocessing vast amounts of soil into fertile "waste" In a single year, as much as 36 tons of soil may pass through the alimentary tracts of all the earthworms living in an acre of soil

71. The word "linked" in the passage is closest in meaning to

A. mechanical B. related C. measured D. chained

72. It could be expected that ants

A. live only above ground B. move more earth than earthworms

C. are more mobile than earthworms D. perform similar functions to fungi

73. The word "bustles" in the passage is closest in meaning to

A. continues B. waits C. is very active D. threatens

74. The main topic of the passage is

A. life in the forest soil B. a description of a forest scene

C. the habits of the forest animals D. the life cycle of ants and worms

75. The soil offers creatures that live underground protection EXPECT

A. extreme heat and cold B. bacteria and fungi C. bad weather D. enemies

76. The word " bulwark" in the passage is closest in meaning to

A. gateway B. barrier C. tomb D. radar

77. It can be inferred from the passage that the forest soil is

A. devoid of life B. densely inhabited C. sparely inhabited D. unknown to man

78. According to the passage, what is the main function of bacteria and fungi?

A. to build walls in the soil B. to help aerate the soil

C. to provide food for plant life D. to kill mature plants

79. The word "constricting" in the passage is closest in meaning to

A. heavy B. deep C. limiting D. damp

80. The word "reprocessing" in the passage is closest in meaning to

(12)

Đáp án đề: 122

01 - | - - 21 { - - - 41 - - } - 61 - - - ~

02 - - - ~ 22 - - - ~ 42 - - } - 62 - - - ~

03 - - - ~ 23 - | - - 43 { - - - 63 - | - -

04 - - - ~ 24 - | - - 44 - - } - 64 - - - ~

05 - - } - 25 - - - ~ 45 - - - ~ 65 - - } -

06 - - - ~ 26 - - - ~ 46 - | - - 66 - | - -

07 { - - - 27 - - - ~ 47 - - } - 67 { - - -

08 - - } - 28 { - - - 48 { - - - 68 - - } -

09 - | - - 29 - - - ~ 49 - - } - 69 - - - ~

10 - - } - 30 - - - ~ 50 - - - ~ 70 { - - -

11 - - - ~ 31 - | - - 51 - - - ~ 71 - | - -

12 { - - - 32 - - - ~ 52 - - } - 72 - - } -

13 - - } - 33 - - } - 53 - | - - 73 - - } -

14 - | - - 34 - | - - 54 - | - - 74 { - - -

15 - - } - 35 - | - - 55 - | - - 75 - | - -

16 - - } - 36 - - } - 56 { - - - 76 - | - -

17 - | - - 37 - - - ~ 57 - - } - 77 - | - -

18 - - - ~ 38 - - } - 58 - - } - 78 - - } -

19 { - - - 39 - | - - 59 - | - - 79 - - } -

(13)

TRƯỜNG THPT LÊ HỒNG PHONG KỲ THI THỬ ĐẠI HỌC - LẦN II Thị xã Bỉm Sơn NĂM HỌC 2010-2011

Môn thi: Tiếng Anh

Thời gian làm bài: 90 phút ĐỀ CHÍNH THỨC

(đề thi gồm trang) Mã đề: 123

Họ tên thí sinh: Số báo danh:

I Read the following passage and mark the letter A, B, C or D on your answer sheet to indicate the correct answer to each of the following questions.

Lead poisoning in children is a major health concern Both low and high doses of lead can have serious effects Children exposed to high doses of lead often suffer permanent nerve damage, mental retardation, short attention spans, distractibility, poor academic performance, and behavioural problems This is not a new concern As early as 1904, lead poisoning in children was linked to lead-based paint Microscopic lead particles from paint are absorbed into bloodstream when the children ingest flakes of chipped paint, plaster or paint dust from suckling, or chewing on toys and other objects painted with lead-based paint Despite amount of lead used in their paint, this source of leading poisoning is still the most common and most dangerous Children living in older, dilapidated houses are particularly at risk

01. What is the main topic of the passage?

A. Lead paint in older houses B. Major health concern for children

C. Problems with household paint D. Lead poisoning in children

02. Which of the following is closest in meaning to the word "suffer" ?

A. reveal B. feel paint from C. grieve with D. experience

03. Which of the following is closest in meaning to "chipped"?

A. sprayed B. fragmented C. unhealthy D. canned

04. The phrase "exposed to" could be best replaced by which of the following?

A. familiar with B. displaying C. in contact with D. conducting

05. According to the passage, what is the most common source of lead poisoning in children?

A. painted toys B. lead-based paint C. dilapidated houses D. household dust

06. Which of the following is NOT true?

A. Only high dose of lead can have serious effects

B. American paint companies today must comply with strict regulations regarding the amount of lead used in their paint

C. Lead poisoning in children was linked to lead-based paint in the 1900s

D. Lead poisoning can lead to mental retardation

07. Which of the following is closest in meaning to the phrase "at risk" ?

A. in danger B. in good condition C. no harm D. in safe way

08. The word " dilapidated " is closest in meaning to which of the following?

A. unpainted B. broken down C. poorly painted D. fell down

09. Which of the following can be inferred from the passage?

A. Paint companies must limit the amount lead used in their paint

B. Paint companies have always followed regulations regarding the amount of lead used in their paint

C. Paint companies can no longer use lead in their paint

D. Paint companies aren't required to limit the amount lead used in their paint

10. What does the author imply in the final sentence of the passage?

A. Poor people did not comply with the regulations

B. Old homes need to be rebuilt in order to be safe for children

C. Old homes were painted with lead-based paint

(14)

II Mark the letter A, B, C, or D on your answer sheet to indicate the correct answer to each of the following sentences

11. I'd sooner deliver the new furniture tomorrow

A. mustn't B. wouldn't C. didn't D. shouldn't

12. Technological advances aid in teaching, the basic role of teachers stays the same

A. despite B. because of C. with D. but

13. As the President was absent, I was ask to the meeting

A. officiate B. chair C. govern D. regulate

14. The product was withdrawn from sale because there was no longer any for it

A. order B. claim C. interest D. call

15. This film several scenes which are very funny

A. pictures B. features C. depicts D. illustrates

16. No one can visit her because she has a very disease

A. contagious B. corporal C. exasperating D. intoxicating

17. How exactly did you set training the horses to work so well together?

A. up B. empty C. loose D. to

18. the public's concern about the local environment this new road scheme will have to be abandoned

A. In view of B. As regards C. In the event of D. However much

19. Bigamy is a situation in which a man two women at the same time

A. marries to B. married C. is marry to D. is married to

20. I'd like to have a bath, but there doesn't seem to be a or anything to stop

A. plug B. cap C. cork D. lid

21. Scientists are still uncertain of originated millions of years ago

A. about the universe B. with the universe C. which the universe D. how the universe

22. There's a rumour that the Nation Bank is going to the company I work for

A. overtake B. take over C. take on D. take off

23. I had to get up early, I'd have missed the train

A. but B. otherwise C. yet D. if not

24. will remain the foremost world language is considered inevitable by many people

A. Because English B. It is English that C. English D. That English

25. "Who broke the window?" - " "

A. I was, thanks B. I did Excuse me C. I did Sorry D. I am

26. All the cereal grains grow on the prairies and plains of the United States

A. but for rice B. but rice C. excepting rice D. except the rice

27. By the year 2012, many people currently employed their jobs

A. will be losing B. have lost C. are losing D. will have lost

28. - Janet: "Do you feel like going to the cinema tonight?" - Susan: " "

A. You're welcome B. I don't agree, I am afraid

C. That would be great D. I feel very bored

29. Only when a great deal of more information has been obtained, to plan a manned trip to the planet

A. it will probably be B. it will be likely C. it will be possible D. will it possible

30. "Don't tell anyone my new address" - " "

A. I can't B. I don't C. I won't D. I wouldn't

31. Automobile propane gas emits fewer dangerous pollutants into the atmosphere

A. using B. that are used C. can use D. use

32. She should have been here but she's flu

A. come up against B. gone through with C. gone down with D. come in for

33. "Can I use your phone?" - " "

A. You must B. No, you can't C. You may not D. Of course

(15)

A. Great you B. Pardon? C. See you D. Bless you

35. A dog on his owner's lap may refuse to eat from a bowl on the floor

A. is fed B. to feed C. was fed D. fed

36. " What's the biggest problem in your country?" - " We have of energy."

A. shortage B. some shortage C. a shortage D. the shortage

37. He tried to explain how he felt, but he was unable to his true feelings

A. evolve B. articulate C. inflict D. inhibit

38. Their climate is not dry at all; in fact, they have of water

A. an abundance B. a deficiency C. a redundancy D. a conglomeration

39. A child in the first grade tends to be all of the other children in the class

A. the same age than B. the same old to C. the same age as D. as old like

40. That popular magazine has many

A. denominations B. subscribers C. prescriptions D. spectators

III Mark the letter A, B, C, or D on your answer sheet to indicate the word that differs from the rest in the position of the main stress in each of the following sentences.

41.A. hotel B. police C. injured D. cassette

42.A. imprudent B. screwdriver C. fortunately D. preferably

43.A. lavatory B. insurance C. government D. envelope

44.A. catalogue B. pedestrian C. experienced D. competitive

45.A. machine B. mistake C. nuclear D. refer

IV Read the following passage and mark the letter A, B, C or D on your answer sheet to indicate the correct word for each of the blanks.

In this age of (46) telephone networks and electronic mail, it seems that fewer and even fewer people are taking time to sit down and write letters (47) friends and relatives For hundreds of years, letters were the only way to keep (48) people who were any distance away and letter-writing was seen as an important skill for all learned people (49) Gradually, (50) , the importance of writing letters is decreasing to a point that majority of us have to (51) a special effort to turn out something worthwhile when we apply for a job or make an appointment In business circles the tendency is for routine communications to become shorter .(52) clients may appreciate a detailed letter, an employee who sends out long letter is often regarded as (53) Many people prefer the telephone in all circumstances and its speed is essential in many situations but (54) have you put the telephone down, dissatisfied with what you have managed to say? I don't think I'll throw my (55) away yet

46.A. all are correct B. highly-developed C. advanced D. progressive

47.A. for B. with C. to D. from

48.A. in contact with B. in favour of C. in step with D. on good terms with

49.A. mastered B. to master C. to be mastered D. mastering

50.A. therefore B. in short C. however D. for example

51.A. cause B. C. create D. make

52.A. Despite B. As though C. However D. Even though

53.A. impossible B. inefficient C. unimportant D. unusual

54.A. how long B. how often C. how much D. how about

55.A. letter B. telephone C. effort D. pen

V Read the following passage and mark the letter A, B, C or D on your answer sheet to indicate the correct answer to each of the following questions.

(16)

downstairs forest denizens live closed linked lives Soil-dwelling bacteria and fungi break down dead organic matter into molecules that above ground plants use for food Those plants as well as animals, mature and die, leaving more organic matter to fuel the folks downstairs

Like a well-insulated house, the soil protects its tenants from extreme temperatures, and from rain and snow It also provides a bulwark against predators that roam the surface world But the dense, protecting soil also limits mobility Soil creatures must be specially equipped in order to travel easily through their dark, constricting realm Earthworms and ants are the champion earth-movers, creating channels that allow air and water to enter the soil While ants travel relatively far from their nests, earthworms work small areas, reprocessing vast amounts of soil into fertile "waste" In a single year, as much as 36 tons of soil may pass through the alimentary tracts of all the earthworms living in an acre of soil

56. It could be expected that ants

A. move more earth than earthworms B. live only above ground

C. are more mobile than earthworms D. perform similar functions to fungi

57. According to the passage, what is the main function of bacteria and fungi?

A. to kill mature plants B. to build walls in the soil

C. to provide food for plant life D. to help aerate the soil

58. The main topic of the passage is

A. the habits of the forest animals B. a description of a forest scene

C. the life cycle of ants and worms D. life in the forest soil

59. The soil offers creatures that live underground protection EXPECT

A. bacteria and fungi B. extreme heat and cold C. bad weather D. enemies

60. The word "linked" in the passage is closest in meaning to

A. mechanical B. chained C. related D. measured

61. It can be inferred from the passage that the forest soil is

A. devoid of life B. densely inhabited C. unknown to man D. sparely inhabited

62. The word "constricting" in the passage is closest in meaning to

A. damp B. heavy C. limiting D. deep

63. The word "bustles" in the passage is closest in meaning to

A. threatens B. is very active C. waits D. continues

64. The word " bulwark" in the passage is closest in meaning to

A. gateway B. barrier C. radar D. tomb

65. The word "reprocessing" in the passage is closest in meaning to

A. eliminating B. converting C. transporting D. arranging VI Mark the letter A, B, C, or D on your answer sheet to show the underlined part that needs correction

66. Vitamin K providing the necessary impetus for the synthesis of at least two proteins involved in blood clotting

A. at least B. necessary C. involved D. providing

67. The nests of most bird species are strategic placed to camouflage them against predators

A. species B. against predators C. strategic D. most

68. For the past few years, researchers have perfecting their control over the movements of cells and microbes by using low power laser beams

A. few years B. their C. using D. have perfecting

69. Perhaps the most unique thing about carbon atoms are their ability to combine with themselves

A. the most B. about C. are D. to combine

70. Organisms and their cells live by maintaining a constant exchange of elemental, ions, minerals and gases

A. gases B. constant C. of elemental D. by maintaining

(17)

71. She has lost her appetite recently

A. She has eaten a lot of food recently B. Her appetite has been very good

C. She hasn't had any food recently D. She has gone off food recently

72. We came home early to avoid the coming storm

A. We didn't come home late to not get the coming storm

B. Because it was predicted storming, we decided to come home earlier than unusual

C. We came home early in order to avoid the coming storm

D. In order to avoiding the coming storm, we came home early

73. "How beautiful is the dress you have just bought!" Peter said to Mary

A. Peter asked Mary how she had just bought her beautiful dress

B. Peter said thanks to Mary for her beautiful dress

C. Peter complimented Mary on her beautiful dress

D. Peter promised to buy Mary a beautiful dress

74. This affair does not concern you

A. Your concern is to this affair B. This affair is no business of yours

C. This affair is not interesting D. Don't this affair

75. She started work three months ago

A. She is working here for three months now B. It's three months that she worked for

C. It is three months since she started work D. She had been working for three months

76. My career as a teacher began 14 years ago

A. For 14 years have I been a teacher B. I was a teacher for 14 years

C. I have been a teacher for 14 years now D. I have started teaching for 14 years now

77. My father regretted working for that company

A. My father wished he hadn't worked for that company

B. If only my father was not working for that company

C. My father didn't like working for that company

D. It is my father's regret working for that company

78. This is the most delicious meat I've ever eaten

A. I have never eaten such more delicious meat

B. Not ever in my life have I had such a wonderful meat

C. At no time in my life have I never tasted this excellent meat

D. Never in my life have I had such a delicious meat

79. My father finds maps hard to follow

A. My father always gets lost B. My father can't read maps at all

C. Map-reading is not interesting to my father at all D. My father has trouble following maps

80. Not many people attended the meeting

A. Too many people turned out at the meeting B. There was a poor turn-out for the meeting

(18)

Đáp án đề: 123

01 - - - ~ 21 - - - ~ 41 { - - - 61 - | - -

02 - - - ~ 22 - | - - 42 { - - - 62 - - } -

03 - | - - 23 - | - - 43 - | - - 63 - | - -

04 - - } - 24 - - - ~ 44 { - - - 64 - | - -

05 - | - - 25 - - } - 45 - - } - 65 - | - -

06 { - - - 26 - | - - 46 { - - - 66 - - - ~

07 { - - - 27 - - - ~ 47 - - } - 67 - - } -

08 - | - - 28 - - } - 48 { - - - 68 - - - ~

09 { - - - 29 - - - ~ 49 - | - - 69 - - } -

10 - | - - 30 - - } - 50 - - } - 70 - - } -

11 - - - ~ 31 { - - - 51 - - - ~ 71 - - - ~

12 - - - ~ 32 - - } - 52 - - - ~ 72 - - } -

13 - | - - 33 - - - ~ 53 - | - - 73 - - } -

14 - - - ~ 34 - - - ~ 54 - | - - 74 - | - -

15 - | - - 35 - - - ~ 55 - - - ~ 75 - - } -

16 { - - - 36 - - } - 56 - - } - 76 - - } -

17 { - - - 37 - | - - 57 - - } - 77 { - - -

18 { - - - 38 { - - - 58 - - - ~ 78 { - - -

19 - - - ~ 39 - - } - 59 { - - - 79 - - - ~

(19)

TRƯỜNG THPT LÊ HỒNG PHONG KỲ THI THỬ ĐẠI HỌC - LẦN II Thị xã Bỉm Sơn NĂM HỌC 2010-2011

Môn thi: Tiếng Anh

Thời gian làm bài: 90 phút ĐỀ CHÍNH THỨC

(đề thi gồm trang) Mã đề: 124

Họ tên thí sinh: Số báo danh:

I Mark the letter A, B, C or D on your answer sheet to indicate the sentence that is closest in meaning to each of following sentences.

01. "How beautiful is the dress you have just bought!" Peter said to Mary

A. Peter asked Mary how she had just bought her beautiful dress

B. Peter promised to buy Mary a beautiful dress

C. Peter complimented Mary on her beautiful dress

D. Peter said thanks to Mary for her beautiful dress

02. My father finds maps hard to follow

A. Map-reading is not interesting to my father at all B. My father can't read maps at all

C. My father has trouble following maps D. My father always gets lost

03. My career as a teacher began 14 years ago

A. I have started teaching for 14 years now B. For 14 years have I been a teacher

C. I have been a teacher for 14 years now D. I was a teacher for 14 years

04. This is the most delicious meat I've ever eaten

A. I have never eaten such more delicious meat

B. Not ever in my life have I had such a wonderful meat

C. At no time in my life have I never tasted this excellent meat

D. Never in my life have I had such a delicious meat

05. This affair does not concern you

A. This affair is no business of yours B. Don't this affair

C. This affair is not interesting D. Your concern is to this affair

06. We came home early to avoid the coming storm

A. We didn't come home late to not get the coming storm

B. In order to avoiding the coming storm, we came home early

C. We came home early in order to avoid the coming storm

D. Because it was predicted storming, we decided to come home earlier than unusual

07. My father regretted working for that company

A. My father wished he hadn't worked for that company

B. My father didn't like working for that company

C. It is my father's regret working for that company

D. If only my father was not working for that company

08. She started work three months ago

A. She had been working for three months B. She is working here for three months now

C. It's three months that she worked for D. It is three months since she started work

09. Not many people attended the meeting

A. People didn't show up at the meeting B. No people came to the meeting

C. There was a poor turn-out for the meeting D. Too many people turned out at the meeting

10. She has lost her appetite recently

A. She hasn't had any food recently B. She has eaten a lot of food recently

C. She has gone off food recently D. Her appetite has been very good

(20)

11. Bigamy is a situation in which a man two women at the same time

A. is married to B. married C. is marry to D. marries to

12. "Don't tell anyone my new address" - " "

A. I don't B. I can't C. I won't D. I wouldn't

13. - Janet: "Do you feel like going to the cinema tonight?" - Susan: " "

A. That would be great B. I don't agree, I am afraid

C. You're welcome D. I feel very bored

14. As the President was absent, I was ask to the meeting

A. regulate B. chair C. govern D. officiate

15. He tried to explain how he felt, but he was unable to his true feelings

A. inhibit B. evolve C. inflict D. articulate

16. A dog on his owner's lap may refuse to eat from a bowl on the floor

A. fed B. was fed C. is fed D. to feed

17. There's a rumour that the Nation Bank is going to the company I work for

A. overtake B. take off C. take over D. take on

18. Scientists are still uncertain of originated millions of years ago

A. with the universe B. about the universe C. how the universe D. which the universe

19. By the year 2012, many people currently employed their jobs

A. will have lost B. are losing C. have lost D. will be losing

20. Their climate is not dry at all; in fact, they have of water

A. an abundance B. a redundancy C. a conglomeration D. a deficiency

21. I had to get up early, I'd have missed the train

A. if not B. but C. yet D. otherwise

22. I'd sooner deliver the new furniture tomorrow

A. wouldn't B. shouldn't C. didn't D. mustn't

23. All the cereal grains grow on the prairies and plains of the United States

A. but for rice B. excepting rice C. except the rice D. but rice

24. the public's concern about the local environment this new road scheme will have to be abandoned

A. In the event of B. However much C. In view of D. As regards

25. will remain the foremost world language is considered inevitable by many people

A. It is English that B. Because English C. English D. That English

26. The product was withdrawn from sale because there was no longer any for it

A. claim B. call C. interest D. order

27. When another person sneezes, you say " "

A. See you B. Bless you C. Great you D. Pardon?

28. She should have been here but she's flu

A. come up against B. gone down with C. gone through with D. come in for

29. A child in the first grade tends to be all of the other children in the class

A. the same age than B. the same old to C. the same age as D. as old like

30. Only when a great deal of more information has been obtained, to plan a manned trip to the planet

A. will it possible B. it will be likely C. it will probably be D. it will be possible

31. Automobile propane gas emits fewer dangerous pollutants into the atmosphere

A. using B. can use C. that are used D. use

32. "Can I use your phone?" - " "

A. No, you can't B. You must C. Of course D. You may not

33. Technological advances aid in teaching, the basic role of teachers stays the same

A. because of B. with C. despite D. but

34. That popular magazine has many

A. spectators B. subscribers C. denominations D. prescriptions

(21)

A. to B. loose C. up D. empty

36. " What's the biggest problem in your country?" - " We have of energy."

A. a shortage B. some shortage C. shortage D. the shortage

37. No one can visit her because she has a very disease

A. corporal B. intoxicating C. contagious D. exasperating

38. I'd like to have a bath, but there doesn't seem to be a or anything to stop

A. cap B. cork C. lid D. plug

39. This film several scenes which are very funny

A. features B. depicts C. illustrates D. pictures

40. "Who broke the window?" - " "

A. I did Excuse me B. I am C. I was, thanks D. I did Sorry III Mark the letter A, B, C, or D on your answer sheet to show the underlined part that needs correction

41. For the past few years, researchers have perfecting their control over the movements of cells and microbes by using low power laser beams

A. few years B. have perfecting C. their D. using

42. Vitamin K providing the necessary impetus for the synthesis of at least two proteins involved in blood clotting

A. at least B. necessary C. involved D. providing

43. Organisms and their cells live by maintaining a constant exchange of elemental, ions, minerals and gases

A. by maintaining B. constant C. gases D. of elemental

44. The nests of most bird species are strategic placed to camouflage them against predators

A. strategic B. against predators C. most D. species

45. Perhaps the most unique thing about carbon atoms are their ability to combine with themselves

A. about B. are C. the most D. to combine

IV Read the following passage and mark the letter A, B, C or D on your answer sheet to indicate the correct word for each of the blanks.

In this age of (46) telephone networks and electronic mail, it seems that fewer and even fewer people are taking time to sit down and write letters (47) friends and relatives For hundreds of years, letters were the only way to keep (48) people who were any distance away and letter-writing was seen as an important skill for all learned people (49) Gradually, (50) , the importance of writing letters is decreasing to a point that majority of us have to (51) a special effort to turn out something worthwhile when we apply for a job or make an appointment In business circles the tendency is for routine communications to become shorter .(52) clients may appreciate a detailed letter, an employee who sends out long letter is often regarded as (53) Many people prefer the telephone in all circumstances and its speed is essential in many situations but (54) have you put the telephone down, dissatisfied with what you have managed to say? I don't think I'll throw my (55) away yet

46.A. all are correct B. advanced C. highly-developed D. progressive

47.A. for B. from C. with D. to

48.A. in step with B. in favour of C. on good terms with D. in contact with

49.A. mastering B. to be mastered C. to master D. mastered

50.A. therefore B. in short C. for example D. however

51.A. create B. C. make D. cause

52.A. Despite B. However C. As though D. Even though

53.A. unimportant B. unusual C. inefficient D. impossible

54.A. how about B. how often C. how much D. how long

(22)

V Mark the letter A, B, C, or D on your answer sheet to indicate the word that differs from the rest in the position of the main stress in each of the following sentences.

56.A. catalogue B. competitive C. experienced D. pedestrian

57.A. screwdriver B. preferably C. imprudent D. fortunately

58.A. mistake B. refer C. machine D. nuclear

59.A. lavatory B. insurance C. envelope D. government

60.A. injured B. hotel C. police D. cassette

VI Read the following passage and mark the letter A, B, C or D on your answer sheet to indicate the correct answer to each of the following questions.

Lead poisoning in children is a major health concern Both low and high doses of lead can have serious effects Children exposed to high doses of lead often suffer permanent nerve damage, mental retardation, short attention spans, distractibility, poor academic performance, and behavioural problems This is not a new concern As early as 1904, lead poisoning in children was linked to lead-based paint Microscopic lead particles from paint are absorbed into bloodstream when the children ingest flakes of chipped paint, plaster or paint dust from suckling, or chewing on toys and other objects painted with lead-based paint Despite amount of lead used in their paint, this source of leading poisoning is still the most common and most dangerous Children living in older, dilapidated houses are particularly at risk

61. What is the main topic of the passage?

A. Lead paint in older houses B. Lead poisoning in children

C. Major health concern for children D. Problems with household paint

62. Which of the following is closest in meaning to "chipped"?

A. unhealthy B. canned C. sprayed D. fragmented

63. What does the author imply in the final sentence of the passage?

A. Poor people did not comply with the regulations

B. Lead-based paint chips off more easily than newer paints

C. Old homes need to be rebuilt in order to be safe for children

D. Old homes were painted with lead-based paint

64. The word " dilapidated " is closest in meaning to which of the following?

A. unpainted B. fell down C. poorly painted D. broken down

65. Which of the following is NOT true?

A. Lead poisoning can lead to mental retardation

B. Lead poisoning in children was linked to lead-based paint in the 1900s

C. Only high dose of lead can have serious effects

D. American paint companies today must comply with strict regulations regarding the amount of lead used in their paint

66. Which of the following is closest in meaning to the word "suffer" ?

A. reveal B. grieve with C. feel paint from D. experience

67. According to the passage, what is the most common source of lead poisoning in children?

A. lead-based paint B. dilapidated houses C. household dust D. painted toys

68. Which of the following can be inferred from the passage?

A. Paint companies must limit the amount lead used in their paint

B. Paint companies have always followed regulations regarding the amount of lead used in their paint

C. Paint companies can no longer use lead in their paint

D. Paint companies aren't required to limit the amount lead used in their paint

69. Which of the following is closest in meaning to the phrase "at risk" ?

A. no harm B. in danger C. in safe way D. in good condition

70. The phrase "exposed to" could be best replaced by which of the following?

A. in contact with B. displaying C. familiar with D. conducting

(23)

Watching for wildlife in the forests, we rarely see past the surface of things Standing on the ground floor, we scan the leafy rafters, entirely overlooking the living world in the soil beneath our feet The forest's basement is a secret world As different from our own world as water is from air, the soil seems quiet, even dead But life bustles down below: a cubic inch of top soil may contain billions of creatures Predators and prey roam beneath as well as above the forest floor Furthermore, those upstairs and downstairs forest denizens live closed linked lives Soil-dwelling bacteria and fungi break down dead organic matter into molecules that above ground plants use for food Those plants as well as animals, mature and die, leaving more organic matter to fuel the folks downstairs

Like a well-insulated house, the soil protects its tenants from extreme temperatures, and from rain and snow It also provides a bulwark against predators that roam the surface world But the dense, protecting soil also limits mobility Soil creatures must be specially equipped in order to travel easily through their dark, constricting realm Earthworms and ants are the champion earth-movers, creating channels that allow air and water to enter the soil While ants travel relatively far from their nests, earthworms work small areas, reprocessing vast amounts of soil into fertile "waste" In a single year, as much as 36 tons of soil may pass through the alimentary tracts of all the earthworms living in an acre of soil

71. It could be expected that ants

A. live only above ground B. are more mobile than earthworms

C. perform similar functions to fungi D. move more earth than earthworms

72. The word "reprocessing" in the passage is closest in meaning to

A. transporting B. converting C. arranging D. eliminating

73. The word " bulwark" in the passage is closest in meaning to

A. tomb B. gateway C. radar D. barrier

74. The main topic of the passage is

A. the life cycle of ants and worms B. life in the forest soil

C. a description of a forest scene D. the habits of the forest animals

75. It can be inferred from the passage that the forest soil is

A. unknown to man B. densely inhabited C. devoid of life D. sparely inhabited

76. The word "bustles" in the passage is closest in meaning to

A. continues B. waits C. threatens D. is very active

77. The word "constricting" in the passage is closest in meaning to

A. limiting B. damp C. deep D. heavy

78. The soil offers creatures that live underground protection EXPECT

A. bad weather B. enemies C. extreme heat and cold D. bacteria and fungi

79. The word "linked" in the passage is closest in meaning to

A. chained B. measured C. related D. mechanical

80. According to the passage, what is the main function of bacteria and fungi?

A. to provide food for plant life B. to build walls in the soil

(24)

Đáp án đề: 124

01 - - } - 21 - - - ~ 41 - | - - 61 - | - -

02 - - } - 22 - | - - 42 - - - ~ 62 - - - ~

03 - - } - 23 - - - ~ 43 - - - ~ 63 - - } -

04 { - - - 24 - - } - 44 { - - - 64 - - - ~

05 { - - - 25 - - - ~ 45 - | - - 65 - - } -

06 - - } - 26 - | - - 46 { - - - 66 - - - ~

07 { - - - 27 - | - - 47 - - - ~ 67 { - - -

08 - - - ~ 28 - | - - 48 - - - ~ 68 { - - -

09 - - } - 29 - - } - 49 - - } - 69 - | - -

10 - - } - 30 { - - - 50 - - - ~ 70 { - - -

11 { - - - 31 { - - - 51 - - } - 71 - | - -

12 - - } - 32 - - } - 52 - - - ~ 72 - | - -

13 { - - - 33 - - - ~ 53 - - } - 73 - - - ~

14 - | - - 34 - | - - 54 - | - - 74 - | - -

15 - - - ~ 35 - - } - 55 { - - - 75 - | - -

16 { - - - 36 { - - - 56 { - - - 76 - - - ~

17 - - } - 37 - - } - 57 - - } - 77 { - - -

18 - - } - 38 - - - ~ 58 - - - ~ 78 - - - ~

19 { - - - 39 { - - - 59 - | - - 79 - - } -

(25)

TRƯỜNG THPT LÊ HỒNG PHONG KỲ THI THỬ ĐẠI HỌC - LẦN II Thị xã Bỉm Sơn NĂM HỌC 2010-2011

Môn thi: Tiếng Anh

Thời gian làm bài: 90 phút ĐỀ CHÍNH THỨC

(đề thi gồm trang) Mã đề: 125

Họ tên thí sinh: Số báo danh:

I Mark the letter A, B, C, or D on your answer sheet to indicate the correct answer to each of the following sentences

01. I'd sooner deliver the new furniture tomorrow

A. mustn't B. didn't C. wouldn't D. shouldn't

02. will remain the foremost world language is considered inevitable by many people

A. English B. That English C. It is English that D. Because English

03. Technological advances aid in teaching, the basic role of teachers stays the same

A. because of B. despite C. but D. with

04. As the President was absent, I was ask to the meeting

A. govern B. officiate C. regulate D. chair

05. Only when a great deal of more information has been obtained, to plan a manned trip to the planet

A. will it possible B. it will be likely C. it will be possible D. it will probably be

06. How exactly did you set training the horses to work so well together?

A. empty B. loose C. up D. to

07. Scientists are still uncertain of originated millions of years ago

A. how the universe B. with the universe C. which the universe D. about the universe

08. " What's the biggest problem in your country?" - " We have of energy."

A. a shortage B. shortage C. some shortage D. the shortage

09. the public's concern about the local environment this new road scheme will have to be abandoned

A. As regards B. In the event of C. However much D. In view of

10. Automobile propane gas emits fewer dangerous pollutants into the atmosphere

A. using B. that are used C. use D. can use

11. All the cereal grains grow on the prairies and plains of the United States

A. except the rice B. but rice C. but for rice D. excepting rice

12. A child in the first grade tends to be all of the other children in the class

A. the same age as B. the same age than C. as old like D. the same old to

13. There's a rumour that the Nation Bank is going to the company I work for

A. overtake B. take over C. take on D. take off

14. A dog on his owner's lap may refuse to eat from a bowl on the floor

A. to feed B. fed C. was fed D. is fed

15. That popular magazine has many

A. subscribers B. spectators C. denominations D. prescriptions

16. When another person sneezes, you say " "

A. Great you B. Pardon? C. See you D. Bless you

17. "Who broke the window?" - " "

A. I am B. I did Sorry C. I was, thanks D. I did Excuse me

18. I'd like to have a bath, but there doesn't seem to be a or anything to stop

A. plug B. lid C. cork D. cap

19. By the year 2012, many people currently employed their jobs

A. will have lost B. have lost C. will be losing D. are losing

(26)

A. I wouldn't B. I don't C. I won't D. I can't

21. I had to get up early, I'd have missed the train

A. otherwise B. yet C. if not D. but

22. Bigamy is a situation in which a man two women at the same time

A. is marry to B. married C. marries to D. is married to

23. He tried to explain how he felt, but he was unable to his true feelings

A. evolve B. inflict C. inhibit D. articulate

24. The product was withdrawn from sale because there was no longer any for it

A. claim B. call C. order D. interest

25. - Janet: "Do you feel like going to the cinema tonight?" - Susan: " "

A. I don't agree, I am afraid B. I feel very bored

C. That would be great D. You're welcome

26. "Can I use your phone?" - " "

A. Of course B. You must C. You may not D. No, you can't

27. No one can visit her because she has a very disease

A. intoxicating B. corporal C. contagious D. exasperating

28. She should have been here but she's flu

A. gone down with B. come in for C. gone through with D. come up against

29. This film several scenes which are very funny

A. depicts B. pictures C. illustrates D. features

30. Their climate is not dry at all; in fact, they have of water

A. an abundance B. a conglomeration C. a redundancy D. a deficiency

II Mark the letter A, B, C, or D on your answer sheet to indicate the word that differs from the rest in the position of the main stress in each of the following sentences.

31.A. competitive B. pedestrian C. catalogue D. experienced

32.A. envelope B. insurance C. government D. lavatory

33.A. fortunately B. imprudent C. preferably D. screwdriver

34.A. nuclear B. mistake C. machine D. refer

35.A. hotel B. cassette C. police D. injured

III Read the following passage and mark the letter A, B, C or D on your answer sheet to indicate the correct answer to each of the following questions.

Lead poisoning in children is a major health concern Both low and high doses of lead can have serious effects Children exposed to high doses of lead often suffer permanent nerve damage, mental retardation, short attention spans, distractibility, poor academic performance, and behavioural problems This is not a new concern As early as 1904, lead poisoning in children was linked to lead-based paint Microscopic lead particles from paint are absorbed into bloodstream when the children ingest flakes of chipped paint, plaster or paint dust from suckling, or chewing on toys and other objects painted with lead-based paint Despite amount of lead used in their paint, this source of leading poisoning is still the most common and most dangerous Children living in older, dilapidated houses are particularly at risk

36. What is the main topic of the passage?

A. Major health concern for children B. Lead poisoning in children

C. Problems with household paint D. Lead paint in older houses

37. What does the author imply in the final sentence of the passage?

A. Poor people did not comply with the regulations

B. Lead-based paint chips off more easily than newer paints

C. Old homes need to be rebuilt in order to be safe for children

D. Old homes were painted with lead-based paint

38. Which of the following is closest in meaning to "chipped"?

A. unhealthy B. sprayed C. canned D. fragmented

(27)

A. Paint companies have always followed regulations regarding the amount of lead used in their paint

B. Paint companies can no longer use lead in their paint

C. Paint companies aren't required to limit the amount lead used in their paint

D. Paint companies must limit the amount lead used in their paint

40. According to the passage, what is the most common source of lead poisoning in children?

A. dilapidated houses B. painted toys C. household dust D. lead-based paint

41. Which of the following is closest in meaning to the phrase "at risk" ?

A. no harm B. in danger C. in safe way D. in good condition

42. Which of the following is NOT true?

A. Lead poisoning can lead to mental retardation

B. Lead poisoning in children was linked to lead-based paint in the 1900s

C. Only high dose of lead can have serious effects

D. American paint companies today must comply with strict regulations regarding the amount of lead used in their paint

43. Which of the following is closest in meaning to the word "suffer" ?

A. grieve with B. feel paint from C. experience D. reveal

44. The phrase "exposed to" could be best replaced by which of the following?

A. familiar with B. in contact with C. conducting D. displaying

45. The word " dilapidated " is closest in meaning to which of the following?

A. broken down B. fell down C. poorly painted D. unpainted

IV Mark the letter A, B, C or D on your answer sheet to indicate the sentence that is closest in meaning to each of following sentences.

46. My career as a teacher began 14 years ago

A. I have been a teacher for 14 years now B. I have started teaching for 14 years now

C. For 14 years have I been a teacher D. I was a teacher for 14 years

47. "How beautiful is the dress you have just bought!" Peter said to Mary

A. Peter said thanks to Mary for her beautiful dress

B. Peter asked Mary how she had just bought her beautiful dress

C. Peter promised to buy Mary a beautiful dress

D. Peter complimented Mary on her beautiful dress

48. This affair does not concern you

A. Don't this affair B. Your concern is to this affair

C. This affair is no business of yours D. This affair is not interesting

49. My father finds maps hard to follow

A. Map-reading is not interesting to my father at all B. My father always gets lost

C. My father has trouble following maps D. My father can't read maps at all

50. Not many people attended the meeting

A. There was a poor turn-out for the meeting B. Too many people turned out at the meeting

C. People didn't show up at the meeting D. No people came to the meeting

51. My father regretted working for that company

A. If only my father was not working for that company

B. It is my father's regret working for that company

C. My father didn't like working for that company

D. My father wished he hadn't worked for that company

52. She started work three months ago

A. She had been working for three months B. She is working here for three months now

C. It is three months since she started work D. It's three months that she worked for

53. We came home early to avoid the coming storm

A. Because it was predicted storming, we decided to come home earlier than unusual

B. In order to avoiding the coming storm, we came home early

(28)

D. We didn't come home late to not get the coming storm

54. This is the most delicious meat I've ever eaten

A. At no time in my life have I never tasted this excellent meat

B. I have never eaten such more delicious meat

C. Not ever in my life have I had such a wonderful meat

D. Never in my life have I had such a delicious meat

55. She has lost her appetite recently

A. She has eaten a lot of food recently B. She has gone off food recently

C. Her appetite has been very good D. She hasn't had any food recently

V Read the following passage and mark the letter A, B, C or D on your answer sheet to indicate the correct word for each of the blanks.

In this age of (56) telephone networks and electronic mail, it seems that fewer and even fewer people are taking time to sit down and write letters (57) friends and relatives For hundreds of years, letters were the only way to keep (58) people who were any distance away and letter-writing was seen as an important skill for all learned people (59) Gradually, (60) , the importance of writing letters is decreasing to a point that majority of us have to (61) a special effort to turn out something worthwhile when we apply for a job or make an appointment In business circles the tendency is for routine communications to become shorter .(62) clients may appreciate a detailed letter, an employee who sends out long letter is often regarded as (63) Many people prefer the telephone in all circumstances and its speed is essential in many situations but (64) have you put the telephone down, dissatisfied with what you have managed to say? I don't think I'll throw my (65) away yet

56.A. all are correct B. highly-developed C. advanced D. progressive

57.A. from B. with C. for D. to

58.A. in step with B. on good terms with C. in favour of D. in contact with

59.A. mastering B. to be mastered C. to master D. mastered

60.A. in short B. however C. therefore D. for example

61.A. create B. C. cause D. make

62.A. As though B. Despite C. Even though D. However

63.A. unimportant B. impossible C. unusual D. inefficient

64.A. how often B. how about C. how long D. how much

65.A. letter B. telephone C. pen D. effort

VI Mark the letter A, B, C, or D on your answer sheet to show the underlined part that needs correction

66. Vitamin K providing the necessary impetus for the synthesis of at least two proteins involved in blood clotting

A. necessary B. providing C. at least D. involved

67. Organisms and their cells live by maintaining a constant exchange of elemental, ions, minerals and gases

A. constant B. of elemental C. gases D. by maintaining

68. Perhaps the most unique thing about carbon atoms are their ability to combine with themselves

A. about B. to combine C. the most D. are

69. For the past few years, researchers have perfecting their control over the movements of cells and microbes by using low power laser beams

A. their B. using C. few years D. have perfecting

70. The nests of most bird species are strategic placed to camouflage them against predators

(29)

VII Read the following passage and mark the letter A, B, C or D on your answer sheet to indicate the correct answer to each of the following questions.

Watching for wildlife in the forests, we rarely see past the surface of things Standing on the ground floor, we scan the leafy rafters, entirely overlooking the living world in the soil beneath our feet The forest's basement is a secret world As different from our own world as water is from air, the soil seems quiet, even dead But life bustles down below: a cubic inch of top soil may contain billions of creatures Predators and prey roam beneath as well as above the forest floor Furthermore, those upstairs and downstairs forest denizens live closed linked lives Soil-dwelling bacteria and fungi break down dead organic matter into molecules that above ground plants use for food Those plants as well as animals, mature and die, leaving more organic matter to fuel the folks downstairs

Like a well-insulated house, the soil protects its tenants from extreme temperatures, and from rain and snow It also provides a bulwark against predators that roam the surface world But the dense, protecting soil also limits mobility Soil creatures must be specially equipped in order to travel easily through their dark, constricting realm Earthworms and ants are the champion earth-movers, creating channels that allow air and water to enter the soil While ants travel relatively far from their nests, earthworms work small areas, reprocessing vast amounts of soil into fertile "waste" In a single year, as much as 36 tons of soil may pass through the alimentary tracts of all the earthworms living in an acre of soil

71. It can be inferred from the passage that the forest soil is

A. unknown to man B. devoid of life C. densely inhabited D. sparely inhabited

72. The word "constricting" in the passage is closest in meaning to

A. damp B. heavy C. limiting D. deep

73. The word "linked" in the passage is closest in meaning to

A. chained B. measured C. related D. mechanical

74. The soil offers creatures that live underground protection EXPECT

A. bad weather B. enemies C. extreme heat and cold D. bacteria and fungi

75. The word "bustles" in the passage is closest in meaning to

A. is very active B. waits C. continues D. threatens

76. According to the passage, what is the main function of bacteria and fungi?

A. to build walls in the soil B. to provide food for plant life

C. to help aerate the soil D. to kill mature plants

77. It could be expected that ants

A. live only above ground B. perform similar functions to fungi

C. are more mobile than earthworms D. move more earth than earthworms

78. The main topic of the passage is

A. life in the forest soil B. the habits of the forest animals

C. the life cycle of ants and worms D. a description of a forest scene

79. The word " bulwark" in the passage is closest in meaning to

A. radar B. tomb C. barrier D. gateway

80. The word "reprocessing" in the passage is closest in meaning to

(30)

Đáp án đề: 125

01 - - - ~ 21 { - - - 41 - | - - 61 - - - ~

02 - | - - 22 - - - ~ 42 - - } - 62 - - } -

03 - - } - 23 - - - ~ 43 - - } - 63 - - - ~

04 - - - ~ 24 - | - - 44 - | - - 64 { - - -

05 { - - - 25 - - } - 45 { - - - 65 - - } -

06 - - } - 26 { - - - 46 { - - - 66 - | - -

07 { - - - 27 - - } - 47 - - - ~ 67 - | - -

08 { - - - 28 { - - - 48 - - } - 68 - - - ~

09 - - - ~ 29 - - - ~ 49 - - } - 69 - - - ~

10 { - - - 30 { - - - 50 { - - - 70 - - - ~

11 - | - - 31 - - } - 51 - - - ~ 71 - - } -

12 { - - - 32 - | - - 52 - - } - 72 - - } -

13 - | - - 33 - | - - 53 - - } - 73 - - } -

14 - | - - 34 { - - - 54 - | - - 74 - - - ~

15 { - - - 35 { - - - 55 - | - - 75 { - - -

16 - - - ~ 36 - | - - 56 { - - - 76 - | - -

17 - | - - 37 - - } - 57 - - - ~ 77 - - } -

18 { - - - 38 - - - ~ 58 - - - ~ 78 { - - -

19 { - - - 39 - - - ~ 59 - - } - 79 - - } -

(31)

TRƯỜNG THPT LÊ HỒNG PHONG KỲ THI THỬ ĐẠI HỌC - LẦN II Thị xã Bỉm Sơn NĂM HỌC 2010-2011

Môn thi: Tiếng Anh

Thời gian làm bài: 90 phút ĐỀ CHÍNH THỨC

(đề thi gồm trang) Mã đề: 12

Họ tên thí sinh: Số báo danh:

I Mark the letter A, B, C, or D on your answer sheet to indicate the word that differs from the rest in the position of the main stress in each of the following sentences.

01.A. preferably B. screwdriver C. fortunately D. imprudent

02.A. competitive B. pedestrian C. experienced D. catalogue

03.A. cassette B. injured C. hotel D. police

04.A. lavatory B. insurance C. government D. envelope

05.A. nuclear B. machine C. refer D. mistake

II Mark the letter A, B, C or D on your answer sheet to indicate the sentence that is closest in meaning to each of following sentences.

06. My father regretted working for that company

A. It is my father's regret working for that company

B. My father wished he hadn't worked for that company

C. My father didn't like working for that company

D. If only my father was not working for that company

07. My career as a teacher began 14 years ago

A. I was a teacher for 14 years B. I have started teaching for 14 years now

C. For 14 years have I been a teacher D. I have been a teacher for 14 years now

08. We came home early to avoid the coming storm

A. In order to avoiding the coming storm, we came home early

B. We came home early in order to avoid the coming storm

C. Because it was predicted storming, we decided to come home earlier than unusual

D. We didn't come home late to not get the coming storm

09. Not many people attended the meeting

A. No people came to the meeting B. There was a poor turn-out for the meeting

C. Too many people turned out at the meeting D. People didn't show up at the meeting

10. This affair does not concern you

A. Don't this affair B. This affair is no business of yours

C. This affair is not interesting D. Your concern is to this affair

11. "How beautiful is the dress you have just bought!" Peter said to Mary

A. Peter said thanks to Mary for her beautiful dress

B. Peter asked Mary how she had just bought her beautiful dress

C. Peter complimented Mary on her beautiful dress

D. Peter promised to buy Mary a beautiful dress

12. My father finds maps hard to follow

A. Map-reading is not interesting to my father at all B. My father always gets lost

C. My father can't read maps at all D. My father has trouble following maps

13. She has lost her appetite recently

A. She has eaten a lot of food recently B. She hasn't had any food recently

C. She has gone off food recently D. Her appetite has been very good

14. She started work three months ago

A. It's three months that she worked for B. It is three months since she started work

(32)

15. This is the most delicious meat I've ever eaten

A. Never in my life have I had such a delicious meat

B. At no time in my life have I never tasted this excellent meat

C. Not ever in my life have I had such a wonderful meat

D. I have never eaten such more delicious meat

III Read the following passage and mark the letter A, B, C or D on your answer sheet to indicate the correct word for each of the blanks.

In this age of (16) telephone networks and electronic mail, it seems that fewer and even fewer people are taking time to sit down and write letters (17) friends and relatives For hundreds of years, letters were the only way to keep (18) people who were any distance away and letter-writing was seen as an important skill for all learned people (19) Gradually, (20) , the importance of writing letters is decreasing to a point that majority of us have to (21) a special effort to turn out something worthwhile when we apply for a job or make an appointment In business circles the tendency is for routine communications to become shorter .(22) clients may appreciate a detailed letter, an employee who sends out long letter is often regarded as (23) Many people prefer the telephone in all circumstances and its speed is essential in many situations but (24) have you put the telephone down, dissatisfied with what you have managed to say? I don't think I'll throw my (25) away yet

16.A. advanced B. all are correct C. progressive D. highly-developed

17.A. from B. to C. for D. with

18.A. in contact with B. in step with C. on good terms with D. in favour of

19.A. to be mastered B. mastering C. mastered D. to master

20.A. for example B. in short C. however D. therefore

21.A. make B. cause C. create D.

22.A. However B. As though C. Despite D. Even though

23.A. inefficient B. unimportant C. impossible D. unusual

24.A. how long B. how often C. how about D. how much

25.A. pen B. letter C. telephone D. effort

IV Mark the letter A, B, C, or D on your answer sheet to indicate the correct answer to each of the following sentences

26. I'd sooner deliver the new furniture tomorrow

A. wouldn't B. didn't C. mustn't D. shouldn't

27. A child in the first grade tends to be all of the other children in the class

A. the same old to B. as old like C. the same age than D. the same age as

28. She should have been here but she's flu

A. gone through with B. come in for C. gone down with D. come up against

29. Only when a great deal of more information has been obtained, to plan a manned trip to the planet

A. will it possible B. it will be possible C. it will be likely D. it will probably be

30. He tried to explain how he felt, but he was unable to his true feelings

A. articulate B. inhibit C. evolve D. inflict

31. No one can visit her because she has a very disease

A. intoxicating B. exasperating C. contagious D. corporal

32. will remain the foremost world language is considered inevitable by many people

A. It is English that B. English C. That English D. Because English

33. the public's concern about the local environment this new road scheme will have to be abandoned

A. In the event of B. However much C. In view of D. As regards

34. By the year 2012, many people currently employed their jobs

A. are losing B. will have lost C. have lost D. will be losing

(33)

A. I did Sorry B. I am C. I was, thanks D. I did Excuse me

36. I'd like to have a bath, but there doesn't seem to be a or anything to stop

A. plug B. cap C. cork D. lid

37. All the cereal grains grow on the prairies and plains of the United States

A. but rice B. excepting rice C. but for rice D. except the rice

38. This film several scenes which are very funny

A. features B. depicts C. pictures D. illustrates

39. I had to get up early, I'd have missed the train

A. yet B. if not C. otherwise D. but

40. "Don't tell anyone my new address" - " "

A. I won't B. I wouldn't C. I don't D. I can't

41. " What's the biggest problem in your country?" - " We have of energy."

A. the shortage B. shortage C. some shortage D. a shortage

42. How exactly did you set training the horses to work so well together?

A. empty B. loose C. up D. to

43. "Can I use your phone?" - " "

A. You may not B. Of course C. No, you can't D. You must

44. Their climate is not dry at all; in fact, they have of water

A. an abundance B. a redundancy C. a deficiency D. a conglomeration

45. The product was withdrawn from sale because there was no longer any for it

A. order B. interest C. call D. claim

46. Automobile propane gas emits fewer dangerous pollutants into the atmosphere

A. can use B. that are used C. using D. use

47. There's a rumour that the Nation Bank is going to the company I work for

A. overtake B. take on C. take off D. take over

48. Bigamy is a situation in which a man two women at the same time

A. marries to B. is marry to C. married D. is married to

49. - Janet: "Do you feel like going to the cinema tonight?" - Susan: " "

A. That would be great B. You're welcome

C. I don't agree, I am afraid D. I feel very bored

50. A dog on his owner's lap may refuse to eat from a bowl on the floor

A. is fed B. to feed C. was fed D. fed

51. Technological advances aid in teaching, the basic role of teachers stays the same

A. because of B. with C. despite D. but

52. As the President was absent, I was ask to the meeting

A. regulate B. officiate C. govern D. chair

53. When another person sneezes, you say " "

A. Pardon? B. Bless you C. Great you D. See you

54. Scientists are still uncertain of originated millions of years ago

A. how the universe B. about the universe C. with the universe D. which the universe

55. That popular magazine has many

A. spectators B. denominations C. prescriptions D. subscribers

V Mark the letter A, B, C, or D on your answer sheet to show the underlined part that needs correction.

56. The nests of most bird species are strategic placed to camouflage them against predators

A. strategic B. species C. against predators D. most

57. Organisms and their cells live by maintaining a constant exchange of elemental, ions, minerals and gases

A. by maintaining B. constant C. of elemental D. gases

58. Perhaps the most unique thing about carbon atoms are their ability to combine with themselves

(34)

59. Vitamin K providing the necessary impetus for the synthesis of at least two proteins involved in blood clotting

A. at least B. providing C. involved D. necessary

60. For the past few years, researchers have perfecting their control over the movements of cells and microbes by using low power laser beams

A. using B. their C. few years D. have perfecting

VI Read the following passage and mark the letter A, B, C or D on your answer sheet to indicate the correct answer to each of the following questions.

Lead poisoning in children is a major health concern Both low and high doses of lead can have serious effects Children exposed to high doses of lead often suffer permanent nerve damage, mental retardation, short attention spans, distractibility, poor academic performance, and behavioural problems This is not a new concern As early as 1904, lead poisoning in children was linked to lead-based paint Microscopic lead particles from paint are absorbed into bloodstream when the children ingest flakes of chipped paint, plaster or paint dust from suckling, or chewing on toys and other objects painted with lead-based paint Despite amount of lead used in their paint, this source of leading poisoning is still the most common and most dangerous Children living in older, dilapidated houses are particularly at risk

61. The phrase "exposed to" could be best replaced by which of the following?

A. displaying B. familiar with C. in contact with D. conducting

62. Which of the following is closest in meaning to "chipped"?

A. canned B. unhealthy C. sprayed D. fragmented

63. Which of the following is closest in meaning to the phrase "at risk" ?

A. in good condition B. no harm C. in danger D. in safe way

64. The word " dilapidated " is closest in meaning to which of the following?

A. poorly painted B. broken down C. fell down D. unpainted

65. What is the main topic of the passage?

A. Major health concern for children B. Lead paint in older houses

C. Lead poisoning in children D. Problems with household paint

66. Which of the following is NOT true?

A. Only high dose of lead can have serious effects

B. Lead poisoning can lead to mental retardation

C. Lead poisoning in children was linked to lead-based paint in the 1900s

D. American paint companies today must comply with strict regulations regarding the amount of lead used in their paint

67. Which of the following is closest in meaning to the word "suffer" ?

A. reveal B. grieve with C. feel paint from D. experience

68. According to the passage, what is the most common source of lead poisoning in children?

A. household dust B. painted toys C. lead-based paint D. dilapidated houses

69. What does the author imply in the final sentence of the passage?

A. Old homes need to be rebuilt in order to be safe for children

B. Poor people did not comply with the regulations

C. Lead-based paint chips off more easily than newer paints

D. Old homes were painted with lead-based paint

70. Which of the following can be inferred from the passage?

A. Paint companies can no longer use lead in their paint

B. Paint companies aren't required to limit the amount lead used in their paint

C. Paint companies have always followed regulations regarding the amount of lead used in their paint

(35)

VII Read the following passage and mark the letter A, B, C or D on your answer sheet to indicate the correct answer to each of the following questions.

Watching for wildlife in the forests, we rarely see past the surface of things Standing on the ground floor, we scan the leafy rafters, entirely overlooking the living world in the soil beneath our feet The forest's basement is a secret world As different from our own world as water is from air, the soil seems quiet, even dead But life bustles down below: a cubic inch of top soil may contain billions of creatures Predators and prey roam beneath as well as above the forest floor Furthermore, those upstairs and downstairs forest denizens live closed linked lives Soil-dwelling bacteria and fungi break down dead organic matter into molecules that above ground plants use for food Those plants as well as animals, mature and die, leaving more organic matter to fuel the folks downstairs

Like a well-insulated house, the soil protects its tenants from extreme temperatures, and from rain and snow It also provides a bulwark against predators that roam the surface world But the dense, protecting soil also limits mobility Soil creatures must be specially equipped in order to travel easily through their dark, constricting realm Earthworms and ants are the champion earth-movers, creating channels that allow air and water to enter the soil While ants travel relatively far from their nests, earthworms work small areas, reprocessing vast amounts of soil into fertile "waste" In a single year, as much as 36 tons of soil may pass through the alimentary tracts of all the earthworms living in an acre of soil

71. The soil offers creatures that live underground protection EXPECT

A. extreme heat and cold B. enemies C. bacteria and fungi D. bad weather

72. It can be inferred from the passage that the forest soil is

A. sparely inhabited B. unknown to man C. densely inhabited D. devoid of life

73. According to the passage, what is the main function of bacteria and fungi?

A. to build walls in the soil B. to kill mature plants

C. to provide food for plant life D. to help aerate the soil

74. The word "bustles" in the passage is closest in meaning to

A. continues B. waits C. is very active D. threatens

75. The word "constricting" in the passage is closest in meaning to

A. limiting B. deep C. damp D. heavy

76. The word "linked" in the passage is closest in meaning to

A. related B. chained C. mechanical D. measured

77. The word " bulwark" in the passage is closest in meaning to

A. barrier B. tomb C. gateway D. radar

78. It could be expected that ants

A. perform similar functions to fungi B. move more earth than earthworms

C. live only above ground D. are more mobile than earthworms

79. The main topic of the passage is

A. a description of a forest scene B. the life cycle of ants and worms

C. the habits of the forest animals D. life in the forest soil

80. The word "reprocessing" in the passage is closest in meaning to

(36)

Đáp án đề: 126

01 - - - ~ 21 { - - - 41 - - - ~ 61 - - } -

02 - - - ~ 22 - - - ~ 42 - - } - 62 - - - ~

03 - - } - 23 { - - - 43 - | - - 63 - - } -

04 - | - - 24 - | - - 44 { - - - 64 - | - -

05 { - - - 25 { - - - 45 - - } - 65 - - } -

06 - | - - 26 - - - ~ 46 - - } - 66 { - - -

07 - - - ~ 27 - - - ~ 47 - - - ~ 67 - - - ~

08 - | - - 28 - - } - 48 - - - ~ 68 - - } -

09 - | - - 29 { - - - 49 { - - - 69 { - - -

10 - | - - 30 { - - - 50 - - - ~ 70 - - - ~

11 - - } - 31 - - } - 51 - - - ~ 71 - - } -

12 - - - ~ 32 - - } - 52 - - - ~ 72 - - } -

13 - - } - 33 - - } - 53 - | - - 73 - - } -

14 - | - - 34 - | - - 54 { - - - 74 - - } -

15 - - - ~ 35 { - - - 55 - - - ~ 75 { - - -

16 - | - - 36 { - - - 56 { - - - 76 { - - -

17 - | - - 37 { - - - 57 - - } - 77 { - - -

18 { - - - 38 { - - - 58 - - } - 78 - - - ~

19 - - - ~ 39 - - } - 59 - | - - 79 - - - ~

(37)

TRƯỜNG THPT LÊ HỒNG PHONG KỲ THI THỬ ĐẠI HỌC - LẦN II Thị xã Bỉm Sơn NĂM HỌC 2010-2011

Môn thi: Tiếng Anh

Thời gian làm bài: 90 phút ĐỀ CHÍNH THỨC

(đề thi gồm trang) Mã đề: 12

Họ tên thí sinh: Số báo danh:

I Mark the letter A, B, C or D on your answer sheet to indicate the sentence that is closest in meaning to each of following sentences.

01. This is the most delicious meat I've ever eaten

A. Not ever in my life have I had such a wonderful meat

B. I have never eaten such more delicious meat

C. Never in my life have I had such a delicious meat

D. At no time in my life have I never tasted this excellent meat

02. We came home early to avoid the coming storm

A. Because it was predicted storming, we decided to come home earlier than unusual

B. We didn't come home late to not get the coming storm

C. In order to avoiding the coming storm, we came home early

D. We came home early in order to avoid the coming storm

03. "How beautiful is the dress you have just bought!" Peter said to Mary

A. Peter complimented Mary on her beautiful dress

B. Peter said thanks to Mary for her beautiful dress

C. Peter asked Mary how she had just bought her beautiful dress

D. Peter promised to buy Mary a beautiful dress

04. Not many people attended the meeting

A. There was a poor turn-out for the meeting B. Too many people turned out at the meeting

C. No people came to the meeting D. People didn't show up at the meeting

05. My father finds maps hard to follow

A. My father always gets lost B. My father can't read maps at all

C. My father has trouble following maps D. Map-reading is not interesting to my father at all

06. This affair does not concern you

A. Don't this affair B. This affair is no business of yours

C. This affair is not interesting D. Your concern is to this affair

07. My father regretted working for that company

A. My father didn't like working for that company

B. It is my father's regret working for that company

C. If only my father was not working for that company

D. My father wished he hadn't worked for that company

08. My career as a teacher began 14 years ago

A. For 14 years have I been a teacher B. I have started teaching for 14 years now

C. I have been a teacher for 14 years now D. I was a teacher for 14 years

09. She started work three months ago

A. It's three months that she worked for B. She had been working for three months

C. She is working here for three months now D. It is three months since she started work

10. She has lost her appetite recently

A. She has gone off food recently B. Her appetite has been very good

C. She has eaten a lot of food recently D. She hasn't had any food recently

(38)

11. That popular magazine has many

A. spectators B. denominations C. prescriptions D. subscribers

12. Only when a great deal of more information has been obtained, to plan a manned trip to the planet

A. it will be possible B. will it possible C. it will probably be D. it will be likely

13. A child in the first grade tends to be all of the other children in the class

A. the same age than B. the same old to C. as old like D. the same age as

14. Their climate is not dry at all; in fact, they have of water

A. a conglomeration B. a redundancy C. a deficiency D. an abundance

15. A dog on his owner's lap may refuse to eat from a bowl on the floor

A. fed B. to feed C. is fed D. was fed

16. "Who broke the window?" - " "

A. I was, thanks B. I did Sorry C. I did Excuse me D. I am

17. I had to get up early, I'd have missed the train

A. but B. otherwise C. yet D. if not

18. "Don't tell anyone my new address" - " "

A. I wouldn't B. I don't C. I won't D. I can't

19. There's a rumour that the Nation Bank is going to the company I work for

A. take over B. overtake C. take off D. take on

20. All the cereal grains grow on the prairies and plains of the United States

A. but rice B. except the rice C. excepting rice D. but for rice

21. " What's the biggest problem in your country?" - " We have of energy."

A. the shortage B. shortage C. a shortage D. some shortage

22. Automobile propane gas emits fewer dangerous pollutants into the atmosphere

A. can use B. use C. using D. that are used

23. She should have been here but she's flu

A. gone through with B. come up against C. come in for D. gone down with

24. The product was withdrawn from sale because there was no longer any for it

A. call B. interest C. order D. claim

25. Scientists are still uncertain of originated millions of years ago

A. about the universe B. which the universe C. with the universe D. how the universe

26. He tried to explain how he felt, but he was unable to his true feelings

A. articulate B. inflict C. inhibit D. evolve

27. I'd like to have a bath, but there doesn't seem to be a or anything to stop

A. plug B. cap C. lid D. cork

28. This film several scenes which are very funny

A. illustrates B. pictures C. features D. depicts

29. How exactly did you set training the horses to work so well together?

A. empty B. to C. loose D. up

30. will remain the foremost world language is considered inevitable by many people

A. English B. That English C. It is English that D. Because English

31. Bigamy is a situation in which a man two women at the same time

A. is married to B. marries to C. is marry to D. married

32. "Can I use your phone?" - " "

A. Of course B. You may not C. No, you can't D. You must

33. When another person sneezes, you say " "

A. Pardon? B. Great you C. Bless you D. See you

34. the public's concern about the local environment this new road scheme will have to be abandoned

A. In the event of B. However much C. In view of D. As regards

35. - Janet: "Do you feel like going to the cinema tonight?" - Susan: " "

(39)

C. I don't agree, I am afraid D. That would be great

36. Technological advances aid in teaching, the basic role of teachers stays the same

A. with B. because of C. but D. despite

37. No one can visit her because she has a very disease

A. exasperating B. contagious C. intoxicating D. corporal

38. As the President was absent, I was ask to the meeting

A. regulate B. govern C. chair D. officiate

39. By the year 2012, many people currently employed their jobs

A. will be losing B. will have lost C. are losing D. have lost

40. I'd sooner deliver the new furniture tomorrow

A. didn't B. shouldn't C. wouldn't D. mustn't

III Mark the letter A, B, C, or D on your answer sheet to show the underlined part that needs correction

41. Organisms and their cells live by maintaining a constant exchange of elemental, ions, minerals and gases

A. constant B. gases C. by maintaining D. of elemental

42. The nests of most bird species are strategic placed to camouflage them against predators

A. species B. against predators C. most D. strategic

43. For the past few years, researchers have perfecting their control over the movements of cells and microbes by using low power laser beams

A. using B. their C. few years D. have perfecting

44. Perhaps the most unique thing about carbon atoms are their ability to combine with themselves

A. the most B. are C. to combine D. about

45. Vitamin K providing the necessary impetus for the synthesis of at least two proteins involved in blood clotting

A. necessary B. providing C. involved D. at least

IV Mark the letter A, B, C, or D on your answer sheet to indicate the word that differs from the rest in the position of the main stress in each of the following sentences.

46.A. competitive B. pedestrian C. experienced D. catalogue

47.A. cassette B. police C. injured D. hotel

48.A. screwdriver B. imprudent C. preferably D. fortunately

49.A. government B. insurance C. envelope D. lavatory

50.A. nuclear B. machine C. refer D. mistake

V Read the following passage and mark the letter A, B, C or D on your answer sheet to indicate the correct answer to each of the following questions.

Lead poisoning in children is a major health concern Both low and high doses of lead can have serious effects Children exposed to high doses of lead often suffer permanent nerve damage, mental retardation, short attention spans, distractibility, poor academic performance, and behavioural problems This is not a new concern As early as 1904, lead poisoning in children was linked to lead-based paint Microscopic lead particles from paint are absorbed into bloodstream when the children ingest flakes of chipped paint, plaster or paint dust from suckling, or chewing on toys and other objects painted with lead-based paint Despite amount of lead used in their paint, this source of leading poisoning is still the most common and most dangerous Children living in older, dilapidated houses are particularly at risk

51. What is the main topic of the passage?

A. Major health concern for children B. Problems with household paint

C. Lead poisoning in children D. Lead paint in older houses

52. Which of the following can be inferred from the passage?

A. Paint companies can no longer use lead in their paint

(40)

C. Paint companies must limit the amount lead used in their paint

D. Paint companies aren't required to limit the amount lead used in their paint

53. According to the passage, what is the most common source of lead poisoning in children?

A. painted toys B. dilapidated houses C. lead-based paint D. household dust

54. Which of the following is NOT true?

A. Lead poisoning in children was linked to lead-based paint in the 1900s

B. American paint companies today must comply with strict regulations regarding the amount of lead used in their paint

C. Only high dose of lead can have serious effects

D. Lead poisoning can lead to mental retardation

55. The word " dilapidated " is closest in meaning to which of the following?

A. fell down B. broken down C. poorly painted D. unpainted

56. The phrase "exposed to" could be best replaced by which of the following?

A. conducting B. in contact with C. familiar with D. displaying

57. Which of the following is closest in meaning to "chipped"?

A. sprayed B. fragmented C. unhealthy D. canned

58. What does the author imply in the final sentence of the passage?

A. Poor people did not comply with the regulations

B. Lead-based paint chips off more easily than newer paints

C. Old homes need to be rebuilt in order to be safe for children

D. Old homes were painted with lead-based paint

59. Which of the following is closest in meaning to the phrase "at risk" ?

A. in danger B. in safe way C. in good condition D. no harm

60. Which of the following is closest in meaning to the word "suffer" ?

A. grieve with B. experience C. feel paint from D. reveal

VI Read the following passage and mark the letter A, B, C or D on your answer sheet to indicate the correct answer to each of the following questions.

Watching for wildlife in the forests, we rarely see past the surface of things Standing on the ground floor, we scan the leafy rafters, entirely overlooking the living world in the soil beneath our feet The forest's basement is a secret world As different from our own world as water is from air, the soil seems quiet, even dead But life bustles down below: a cubic inch of top soil may contain billions of creatures Predators and prey roam beneath as well as above the forest floor Furthermore, those upstairs and downstairs forest denizens live closed linked lives Soil-dwelling bacteria and fungi break down dead organic matter into molecules that above ground plants use for food Those plants as well as animals, mature and die, leaving more organic matter to fuel the folks downstairs

Like a well-insulated house, the soil protects its tenants from extreme temperatures, and from rain and snow It also provides a bulwark against predators that roam the surface world But the dense, protecting soil also limits mobility Soil creatures must be specially equipped in order to travel easily through their dark, constricting realm Earthworms and ants are the champion earth-movers, creating channels that allow air and water to enter the soil While ants travel relatively far from their nests, earthworms work small areas, reprocessing vast amounts of soil into fertile "waste" In a single year, as much as 36 tons of soil may pass through the alimentary tracts of all the earthworms living in an acre of soil

61. It can be inferred from the passage that the forest soil is

A. unknown to man B. densely inhabited C. sparely inhabited D. devoid of life

62. The word "bustles" in the passage is closest in meaning to

A. continues B. waits C. threatens D. is very active

63. The main topic of the passage is

A. a description of a forest scene B. the life cycle of ants and worms

C. the habits of the forest animals D. life in the forest soil

64. The soil offers creatures that live underground protection EXPECT

(41)

65. The word "reprocessing" in the passage is closest in meaning to

A. transporting B. eliminating C. arranging D. converting

66. The word " bulwark" in the passage is closest in meaning to

A. tomb B. gateway C. barrier D. radar

67. The word "constricting" in the passage is closest in meaning to

A. damp B. deep C. heavy D. limiting

68. It could be expected that ants

A. perform similar functions to fungi B. move more earth than earthworms

C. live only above ground D. are more mobile than earthworms

69. The word "linked" in the passage is closest in meaning to

A. related B. mechanical C. measured D. chained

70. According to the passage, what is the main function of bacteria and fungi?

A. to kill mature plants B. to provide food for plant life

C. to help aerate the soil D. to build walls in the soil

VII Read the following passage and mark the letter A, B, C or D on your answer sheet to indicate the correct word for each of the blanks.

In this age of (71) telephone networks and electronic mail, it seems that fewer and even fewer people are taking time to sit down and write letters (72) friends and relatives For hundreds of years, letters were the only way to keep (73) people who were any distance away and letter-writing was seen as an important skill for all learned people (74) Gradually, (75) , the importance of writing letters is decreasing to a point that majority of us have to (76) a special effort to turn out something worthwhile when we apply for a job or make an appointment In business circles the tendency is for routine communications to become shorter .(77) clients may appreciate a detailed letter, an employee who sends out long letter is often regarded as (78) Many people prefer the telephone in all circumstances and its speed is essential in many situations but (79) have you put the telephone down, dissatisfied with what you have managed to say? I don't think I'll throw my (80) away yet

71.A. all are correct B. highly-developed C. advanced D. progressive

72.A. to B. with C. from D. for

73.A. in step with B. in favour of C. on good terms with D. in contact with

74.A. mastering B. mastered C. to master D. to be mastered

75.A. for example B. therefore C. however D. in short

76.A. cause B. create C. make D.

77.A. However B. As though C. Even though D. Despite

78.A. impossible B. unimportant C. inefficient D. unusual

79.A. how much B. how often C. how about D. how long

(42)

Đáp án đề: 127

01 - | - - 21 - - } - 41 - - - ~ 61 - | - -

02 - - - ~ 22 - - } - 42 - - - ~ 62 - - - ~

03 { - - - 23 - - - ~ 43 - - - ~ 63 - - - ~

04 { - - - 24 { - - - 44 - | - - 64 - | - -

05 - - } - 25 - - - ~ 45 - | - - 65 - - - ~

06 - | - - 26 { - - - 46 - - - ~ 66 - - } -

07 - - - ~ 27 { - - - 47 - - - ~ 67 - - - ~

08 - - } - 28 - - } - 48 - | - - 68 - - - ~

09 - - - ~ 29 - - - ~ 49 - | - - 69 { - - -

10 { - - - 30 - | - - 50 { - - - 70 - | - -

11 - - - ~ 31 { - - - 51 - - } - 71 { - - -

12 - | - - 32 { - - - 52 - - } - 72 { - - -

13 - - - ~ 33 - - } - 53 - - } - 73 - - - ~

14 - - - ~ 34 - - } - 54 - - } - 74 - - } -

15 { - - - 35 - - - ~ 55 - | - - 75 - - } -

16 - | - - 36 - - } - 56 - | - - 76 - - } -

17 - | - - 37 - | - - 57 - | - - 77 - - } -

18 - - } - 38 - - } - 58 - - } - 78 - - } -

19 { - - - 39 - | - - 59 { - - - 79 - | - -

(43)

TRƯỜNG THPT LÊ HỒNG PHONG KỲ THI THỬ ĐẠI HỌC - LẦN II Thị xã Bỉm Sơn NĂM HỌC 2010-2011

Môn thi: Tiếng Anh

Thời gian làm bài: 90 phút ĐỀ CHÍNH THỨC

(đề thi gồm trang) Mã đề: 128

Họ tên thí sinh: Số báo danh:

I Mark the letter A, B, C, or D on your answer sheet to indicate the word that differs from the rest in the position of the main stress in each of the following sentences.

01.A. lavatory B. envelope C. insurance D. government

02.A. catalogue B. experienced C. competitive D. pedestrian

03.A. refer B. mistake C. machine D. nuclear

04.A. injured B. hotel C. police D. cassette

05.A. fortunately B. imprudent C. preferably D. screwdriver

II Read the following passage and mark the letter A, B, C or D on your answer sheet to indicate the correct word for each of the blanks.

In this age of (6) telephone networks and electronic mail, it seems that fewer and even fewer people are taking time to sit down and write letters (7) friends and relatives For hundreds of years, letters were the only way to keep (8) people who were any distance away and letter-writing was seen as an important skill for all learned people (9) Gradually, (10) , the importance of writing letters is decreasing to a point that majority of us have to (11) a special effort to turn out something worthwhile when we apply for a job or make an appointment In business circles the tendency is for routine communications to become shorter .(12) clients may appreciate a detailed letter, an employee who sends out long letter is often regarded as (13) Many people prefer the telephone in all circumstances and its speed is essential in many situations but (14) have you put the telephone down, dissatisfied with what you have managed to say? I don't think I'll throw my (15) away yet

06.A. highly-developed B. advanced C. all are correct D. progressive

07.A. with B. from C. to D. for

08.A. on good terms with B. in favour of C. in step with D. in contact with

09.A. to master B. mastered C. mastering D. to be mastered

10.A. in short B. however C. for example D. therefore

11.A. cause B. create C. make D.

12.A. Despite B. As though C. Even though D. However

13.A. inefficient B. unimportant C. unusual D. impossible

14.A. how long B. how about C. how much D. how often

15.A. effort B. letter C. pen D. telephone

III Mark the letter A, B, C, or D on your answer sheet to show the underlined part that needs correction

16. Perhaps the most unique thing about carbon atoms are their ability to combine with themselves

A. are B. to combine C. the most D. about

17. For the past few years, researchers have perfecting their control over the movements of cells and microbes by using low power laser beams

A. have perfecting B. few years C. using D. their

18. Vitamin K providing the necessary impetus for the synthesis of at least two proteins involved in blood clotting

A. necessary B. providing C. at least D. involved

19. The nests of most bird species are strategic placed to camouflage them against predators

(44)

20. Organisms and their cells live by maintaining a constant exchange of elemental, ions, minerals and gases

A. by maintaining B. gases C. constant D. of elemental

IV Read the following passage and mark the letter A, B, C or D on your answer sheet to indicate the correct answer to each of the following questions.

Lead poisoning in children is a major health concern Both low and high doses of lead can have serious effects Children exposed to high doses of lead often suffer permanent nerve damage, mental retardation, short attention spans, distractibility, poor academic performance, and behavioural problems This is not a new concern As early as 1904, lead poisoning in children was linked to lead-based paint Microscopic lead particles from paint are absorbed into bloodstream when the children ingest flakes of chipped paint, plaster or paint dust from suckling, or chewing on toys and other objects painted with lead-based paint Despite amount of lead used in their paint, this source of leading poisoning is still the most common and most dangerous Children living in older, dilapidated houses are particularly at risk

21. The phrase "exposed to" could be best replaced by which of the following?

A. conducting B. familiar with C. in contact with D. displaying

22. What does the author imply in the final sentence of the passage?

A. Old homes were painted with lead-based paint

B. Old homes need to be rebuilt in order to be safe for children

C. Poor people did not comply with the regulations

D. Lead-based paint chips off more easily than newer paints

23. Which of the following is NOT true?

A. Only high dose of lead can have serious effects

B. Lead poisoning can lead to mental retardation

C. Lead poisoning in children was linked to lead-based paint in the 1900s

D. American paint companies today must comply with strict regulations regarding the amount of lead used in their paint

24. What is the main topic of the passage?

A. Major health concern for children B. Lead poisoning in children

C. Lead paint in older houses D. Problems with household paint

25. Which of the following is closest in meaning to "chipped"?

A. unhealthy B. sprayed C. canned D. fragmented

26. The word " dilapidated " is closest in meaning to which of the following?

A. unpainted B. broken down C. fell down D. poorly painted

27. Which of the following is closest in meaning to the phrase "at risk" ?

A. in good condition B. no harm C. in safe way D. in danger

28. According to the passage, what is the most common source of lead poisoning in children?

A. lead-based paint B. painted toys C. dilapidated houses D. household dust

29. Which of the following can be inferred from the passage?

A. Paint companies must limit the amount lead used in their paint

B. Paint companies aren't required to limit the amount lead used in their paint

C. Paint companies can no longer use lead in their paint

D. Paint companies have always followed regulations regarding the amount of lead used in their paint

30. Which of the following is closest in meaning to the word "suffer" ?

A. feel paint from B. experience C. reveal D. grieve with

V.Mark the letter A, B, C or D on your answer sheet to indicate the sentence that is closest in meaning to each of following sentences.

31. She started work three months ago

A. It's three months that she worked for B. She had been working for three months

C. She is working here for three months now D. It is three months since she started work

32. This is the most delicious meat I've ever eaten

(45)

B. Not ever in my life have I had such a wonderful meat

C. Never in my life have I had such a delicious meat

D. I have never eaten such more delicious meat

33. "How beautiful is the dress you have just bought!" Peter said to Mary

A. Peter complimented Mary on her beautiful dress

B. Peter said thanks to Mary for her beautiful dress

C. Peter promised to buy Mary a beautiful dress

D. Peter asked Mary how she had just bought her beautiful dress

34. We came home early to avoid the coming storm

A. Because it was predicted storming, we decided to come home earlier than unusual

B. We didn't come home late to not get the coming storm

C. In order to avoiding the coming storm, we came home early

D. We came home early in order to avoid the coming storm

35. My career as a teacher began 14 years ago

A. I was a teacher for 14 years B. I have been a teacher for 14 years now

C. I have started teaching for 14 years now D. For 14 years have I been a teacher

36. My father regretted working for that company

A. My father wished he hadn't worked for that company

B. It is my father's regret working for that company

C. My father didn't like working for that company

D. If only my father was not working for that company

37. This affair does not concern you

A. Don't this affair B. Your concern is to this affair

C. This affair is no business of yours D. This affair is not interesting

38. Not many people attended the meeting

A. Too many people turned out at the meeting B. No people came to the meeting

C. People didn't show up at the meeting D. There was a poor turn-out for the meeting

39. My father finds maps hard to follow

A. Map-reading is not interesting to my father at all

B. My father always gets lost

C. My father has trouble following maps

D. My father can't read maps at all

40. She has lost her appetite recently

A. She has gone off food recently B. Her appetite has been very good

C. She has eaten a lot of food recently D. She hasn't had any food recently

VI Mark the letter A, B, C, or D on your answer sheet to indicate the correct answer to each of the following sentences

41. I'd sooner deliver the new furniture tomorrow

A. didn't B. mustn't C. wouldn't D. shouldn't

42. Scientists are still uncertain of originated millions of years ago

A. with the universe B. about the universe C. how the universe D. which the universe

43. She should have been here but she's flu

A. come in for B. gone through with C. gone down with D. come up against

44. How exactly did you set training the horses to work so well together?

A. up B. to C. empty D. loose

45. As the President was absent, I was ask to the meeting

A. officiate B. chair C. regulate D. govern

46. All the cereal grains grow on the prairies and plains of the United States

A. excepting rice B. but for rice C. except the rice D. but rice

47. I'd like to have a bath, but there doesn't seem to be a or anything to stop

(46)

48. "Who broke the window?" - " "

A. I am B. I did Sorry C. I was, thanks D. I did Excuse me

49. " What's the biggest problem in your country?" - " We have of energy."

A. shortage B. a shortage C. the shortage D. some shortage

50. This film several scenes which are very funny

A. illustrates B. features C. depicts D. pictures

51. Only when a great deal of more information has been obtained, to plan a manned trip to the planet

A. will it possible B. it will be possible C. it will be likely D. it will probably be

52. There's a rumour that the Nation Bank is going to the company I work for

A. overtake B. take off C. take over D. take on

53. Technological advances aid in teaching, the basic role of teachers stays the same

A. with B. because of C. despite D. but

54. will remain the foremost world language is considered inevitable by many people

A. That English B. English C. Because English D. It is English that

55. "Can I use your phone?" - " "

A. Of course B. You must C. No, you can't D. You may not

56. Bigamy is a situation in which a man two women at the same time

A. is marry to B. married C. is married to D. marries to

57. A child in the first grade tends to be all of the other children in the class

A. the same old to B. the same age than C. the same age as D. as old like

58. Automobile propane gas emits fewer dangerous pollutants into the atmosphere

A. using B. use C. can use D. that are used

59. The product was withdrawn from sale because there was no longer any for it

A. call B. interest C. order D. claim

60. "Don't tell anyone my new address" - " "

A. I don't B. I wouldn't C. I can't D. I won't

61. Their climate is not dry at all; in fact, they have of water

A. a redundancy B. an abundance C. a deficiency D. a conglomeration

62. I had to get up early, I'd have missed the train

A. otherwise B. but C. if not D. yet

63. When another person sneezes, you say " "

A. See you B. Pardon? C. Great you D. Bless you

64. He tried to explain how he felt, but he was unable to his true feelings

A. evolve B. inhibit C. articulate D. inflict

65. No one can visit her because she has a very disease

A. corporal B. intoxicating C. exasperating D. contagious

66. A dog on his owner's lap may refuse to eat from a bowl on the floor

A. fed B. to feed C. was fed D. is fed

67. the public's concern about the local environment this new road scheme will have to be abandoned

A. However much B. In view of C. In the event of D. As regards

68. - Janet: "Do you feel like going to the cinema tonight?" - Susan: " "

A. I feel very bored B. I don't agree, I am afraid

C. That would be great D. You're welcome

69. By the year 2012, many people currently employed their jobs

A. will be losing B. are losing C. will have lost D. have lost

70. That popular magazine has many

(47)

VII Read the following passage and mark the letter A, B, C or D on your answer sheet to indicate the correct answer to each of the following questions.

Watching for wildlife in the forests, we rarely see past the surface of things Standing on the ground floor, we scan the leafy rafters, entirely overlooking the living world in the soil beneath our feet The forest's basement is a secret world As different from our own world as water is from air, the soil seems quiet, even dead But life bustles down below: a cubic inch of top soil may contain billions of creatures Predators and prey roam beneath as well as above the forest floor Furthermore, those upstairs and downstairs forest denizens live closed linked lives Soil-dwelling bacteria and fungi break down dead organic matter into molecules that above ground plants use for food Those plants as well as animals, mature and die, leaving more organic matter to fuel the folks downstairs

Like a well-insulated house, the soil protects its tenants from extreme temperatures, and from rain and snow It also provides a bulwark against predators that roam the surface world But the dense, protecting soil also limits mobility Soil creatures must be specially equipped in order to travel easily through their dark, constricting realm Earthworms and ants are the champion earth-movers, creating channels that allow air and water to enter the soil While ants travel relatively far from their nests, earthworms work small areas, reprocessing vast amounts of soil into fertile "waste" In a single year, as much as 36 tons of soil may pass through the alimentary tracts of all the earthworms living in an acre of soil

71. The main topic of the passage is

A. life in the forest soil B. a description of a forest scene

C. the habits of the forest animals D. the life cycle of ants and worms

72. It could be expected that ants

A. move more earth than earthworms B. are more mobile than earthworms

C. live only above ground D. perform similar functions to fungi

73. The word "linked" in the passage is closest in meaning to

A. mechanical B. related C. chained D. measured

74. It can be inferred from the passage that the forest soil is

A. devoid of life B. unknown to man C. sparely inhabited D. densely inhabited

75. The soil offers creatures that live underground protection EXPECT

A. bad weather B. bacteria and fungi C. enemies D. extreme heat and cold

76. According to the passage, what is the main function of bacteria and fungi?

A. to help aerate the soil B. to kill mature plants

C. to build walls in the soil D. to provide food for plant life

77. The word " bulwark" in the passage is closest in meaning to

A. barrier B. radar C. gateway D. tomb

78. The word "reprocessing" in the passage is closest in meaning to

A. arranging B. converting C. eliminating D. transporting

79. The word "constricting" in the passage is closest in meaning to

A. deep B. damp C. limiting D. heavy

80. The word "bustles" in the passage is closest in meaning to

(48)

Đáp án đề: 128

01 - - } - 21 - - } - 41 - - - ~ 61 - | - -

02 { - - - 22 - | - - 42 - - } - 62 { - - -

03 - - - ~ 23 { - - - 43 - - } - 63 - - - ~

04 - | - - 24 - | - - 44 { - - - 64 - - } -

05 - | - - 25 - - - ~ 45 - | - - 65 - - - ~

06 - - } - 26 - | - - 46 - - - ~ 66 { - - -

07 - - } - 27 - - - ~ 47 { - - - 67 - | - -

08 - - - ~ 28 { - - - 48 - | - - 68 - - } -

09 { - - - 29 { - - - 49 - | - - 69 - - } -

10 - | - - 30 - | - - 50 - | - - 70 - - } -

11 - - } - 31 - - - ~ 51 { - - - 71 { - - -

12 - - } - 32 - - - ~ 52 - - } - 72 - | - -

13 { - - - 33 { - - - 53 - - - ~ 73 - | - -

14 - - - ~ 34 - - - ~ 54 { - - - 74 - - - ~

15 - - } - 35 - | - - 55 { - - - 75 - | - -

16 { - - - 36 { - - - 56 - - } - 76 - - - ~

17 { - - - 37 - - } - 57 - - } - 77 { - - -

18 - | - - 38 - - - ~ 58 { - - - 78 - | - -

19 - - - ~ 39 - - } - 59 { - - - 79 - - } -

Ngày đăng: 16/05/2021, 14:58

Tài liệu cùng người dùng

  • Đang cập nhật ...

Tài liệu liên quan